6

You might also like

Download as txt, pdf, or txt
Download as txt, pdf, or txt
You are on page 1of 44

1.

Question
The nurse is caring for an elderly woman who has had a fractured hip repaired. In
the first few days following the surgical repair, which of the following nursing
measures will best facilitate the resumption of activities for this client?

A. Arranging for the wheelchair


B. Asking her family to visit
C. Assisting her to sit out of bed in a chair qid
D. Encouraging the use of an overhead trapeze
Incorrect
Correct Answer: D. Encouraging the use of an overhead trapeze.

Exercise is important to keep the joints and muscles functioning and to prevent
secondary complications. Using the overhead trapeze prevents hazards of immobility
by permitting movement in bed and strengthening of the upper extremities in
preparation for ambulation. Facilitates movement during hygiene or skincare and
linen changes; reduces the discomfort of remaining flat in bed. “Post position”
involves placing the uninjured foot flat on the bed with the knee bent while
grasping the trapeze and lifting the body off the bed.

Option A: Sitting in a wheelchair would require too great hip flexion initially.
Place in supine position periodically if possible, when traction is used to
stabilize lower limb fractures. Reduces the risk of flexion contracture of the hip.
Option B: Asking her family to visit would not facilitate the resumption of
activities. Provide footboard, wrist splints, trochanter, or hand rolls as
appropriate. Useful in maintaining a functional position of extremities, hands, and
feet, and preventing complications (contractures, foot drop).
Option C: Sitting in a chair would cause too much hip flexion. The client initially
needs to be in a low Fowler’s position or taking a few steps (as ordered) with the
aid of a walker. Encourage the use of isometric exercises starting with the
unaffected limb. Isometrics contract muscles without bending joints or moving limbs
and help maintain muscle strength and mass. Note: These exercises are
contraindicated while acute bleeding and edema are present.
2. Question
Which of the following is the most important nursing order in a client with major
head trauma who is about to receive bolus enteral feeding?

A. Measure intake and output


B. Check albumin level
C. Monitor glucose levels
D. Increase enteral feeding
Incorrect
Correct Answer: A. Measure intake and output

It is important to measure intake and output, which should be equal. Water given
before feeding will present a hyperosmotic diuresis. I and O measures assess fluid
balance. A urinary catheter is inserted to assess the adequacy of renal perfusion.
The kidney requires 20% to 25% of cardiac output; commonly, it’s the first organ to
show the effects of impaired perfusion or intravascular volume.

Option B: Osmotherapy aims to increase the osmolality of the intravascular space,


which in turn helps mobilize excess fluid from brain tissue. If ICP increases,
mannitol (an osmotic diuretic) may be given to decrease cerebral edema, transiently
increase intravascular volume, and improve cerebral blood flow.
Option C: Low peripheral oxygen saturation values or low arterial blood oxygen
values (as shown by arterial blood gas testing) should be avoided. Maintaining
adequate brain tissue oxygenation seems to improve patient outcomes.
Option D: Enteral feedings are hyperosmotic agents pulling fluid from cells into
the vascular bed. Initially, a nasogastric or orogastric tube is inserted to
decompress the stomach and reduce the aspiration risk. (Typically, the nasal route
is avoided as it can obstruct sinus drainage, leading to sinusitis or VAP).
3. Question
The pathological process causing esophageal varices is/are:

A. Ascites and edema


B. Systemic hypertension
C. Portal hypertension
D. Dilated veins and varicosities
Incorrect
Correct Answer: C. Portal hypertension

Esophageal varices result from increased portal hypertension. In portal


hypertension, the liver cannot accept all of the fluid from the portal vein. The
excess fluid will backflow to the vessels with lesser pressure, such as esophageal
veins or rectal veins causing esophageal varices or hemorrhoids.

Option A: Portal hypertension causes portocaval anastomosis to develop to


decompress portal circulation. Normal portal pressure is between 5-10 mmHg but in
the presence of portal obstruction, the pressure may be as high as 15-20 mmHg.
Since the portal venous system has no valves, resistance at any level between the
splanchnic vessels and the right side of the heart results in retrograde flow and
elevated pressure.
Option B: Esophageal varices are dilated submucosal distal esophageal veins
connecting the portal and systemic circulations. They form due to portal
hypertension, which commonly is a result of cirrhosis, resistance to portal blood
flow, and increased portal venous blood inflow.
Option D: Intrahepatic vasoconstriction due to decreased nitric oxide production,
and increased release of endothelin-1 (ET-1), angiotensinogen, and eicosanoids.
Increased portal flow is caused by hyperdynamic circulation due to splanchnic
arterial vasodilation through mediators such as nitric oxide, prostacyclin, and
TNF.
4. Question
Which of the following interventions will help lessen the effect of GERD (acid
reflux)?

A. Elevate the head of the bed on 4-6 inch blocks.


B. Lie down after eating.
C. Increase fluid intake just before bedtime.
D. Wear a girdle.
Incorrect
Correct Answer: A. Elevate the head of the bed on 4-6 inch blocks.

Elevation of the head of the bed allows gravity to assist in decreasing the
backflow of acid into the esophagus. The fluid does not flow uphill. Instruct to
remain in an upright position at least 2 hours after meals; avoiding eating 3 hours
before bedtime. Helps control reflux and causes less irritation from reflux action
into the esophagus. The other three options all increase fluid backflow into the
esophagus through position or increasing abdominal pressure.

Option B: Avoid placing the patient in a supine position, have the patient sit
upright after meals. Supine position after meals can increase regurgitation of
acid. Elevate HOB while in bed. To prevent aspiration by preventing the gastric
acid to flow back into the esophagus.
Option C: Instruct patient regarding eating small amounts of bland food followed by
a small amount of water. Instruct to remain in an upright position at least 1–2
hours after meals, and to avoid eating within 2–4 hours of bedtime. Gravity helps
control reflux and causes less irritation from reflux action into the esophagus.
Option D: Instruct the patient to avoid bending over, coughing, straining at
defecations, and other activities that increase reflux. Promotes comfort by the
decrease in intra-abdominal pressure, which reduces the reflux of gastric contents.
5. Question
The main benefit of therapeutic massages is:

A. To help a person with swollen legs to decrease fluid retention.


B. To help a person with duodenal ulcers feel better.
C. To help damaged tissue in a diabetic to heal.
D. To improve circulation and muscle tone.
Incorrect
Correct Answer: D. To improve circulation and muscle tone.

Particularly in elderly adults, therapeutic massage will help improve circulation


and muscle tone as well as the personal attention and social interaction that a
good massage provides. Damaged or strained muscle fibers release inflammatory
chemicals to aid the healing process, but these chemicals cause significant pain
and discomfort in the process. At least one study, which looked at the effects of
massage on post-exercise tissue inflammation, suggests that even 10 minutes of
massage can reduce signs of inflammation and improve cell processes, thereby
promoting healing, with effects lasting several hours after the massage.

Option A: Massage only the hands, feet, or scalp of patients with sepsis, fever
over 100[degrees]F, nausea or vomiting, sickle cell crisis, HIV crisis, a
complicated or high-risk pregnancy, crepitus, edema, thrombocytopenia, or
meningitis.
Option B: When patients have fragile skin, or the potential for skin breakdown,
apply only light pressure, using enough lotion or oil to minimize friction. For
patients with a previous injury, chronic pain, or scar tissue, frequently ask them
how the massage feels, and adjust both pressure and massage technique to the
patients’ preferences.
Option C: A massage is contraindicated in any condition where massage to damaged
tissue can dislodge a blood clot. Although massage is associated with few adverse
effects, nurses should be careful to avoid areas near open wounds, any stage of
pressure ulcer, reddened or swollen areas, rashes, incisions, thromboses, iv lines,
drains, shunts, and tubes.
6. Question
Which of the following foods should be avoided by clients who are prone to develop
heartburn as a result of gastroesophageal reflux disease (GERD)?

A. Lettuce
B. Eggs
C. Chocolate
D. Butterscotch
Incorrect
Correct Answer: C. Chocolate

Ingestion of chocolate can reduce lower esophageal sphincter (LES) pressure leading
to reflux and clinical symptoms of GERD. Ingesting cocoa can cause a surge of
serotonin. This surge can cause the esophageal sphincter to relax and gastric
contents to rise. Caffeine and theobromine in chocolate may also trigger acid
reflux. All of the other foods do not affect LES pressure.

Option A: Vegetables are naturally low in fat and sugar, and they help reduce
stomach acid. Good options include green beans, broccoli, asparagus, cauliflower,
leafy greens, potatoes, and cucumbers.
Option B: Egg whites are a good option. Stay away from egg yolks, though, which are
high in fat and may trigger reflux symptoms. Reflux symptoms may result from
stomach acid touching the esophagus and causing irritation and pain.
Option D: The foods the patient eats affect the amount of acid the stomach
produces. Eating the right kinds of food is key to controlling acid reflux or GERD,
a severe, chronic form of acid reflux. Sources of healthy fats include avocados,
walnuts, flaxseed, olive oil, sesame oil, and sunflower oil. Reduce the intake of
saturated fats and trans fats and replace them with these healthier unsaturated
fats.
7. Question
Which of the following should be included in a plan of care for a client receiving
total parenteral nutrition (TPN)?

A. Withhold medications while the TPN is infusing.


B. Change TPN solution every 24 hours.
C. Flush the TPN line with water prior to initiating nutritional support.
D. Keep the client on complete bed rest during TPN therapy.
Incorrect
Correct Answer: B. Change TPN solution every 24 hours.

TPN solutions should be changed every 24 hours in order to prevent bacterial


overgrowth due to the hypertonicity of the solution. Because the central venous
catheter needs to remain in place for a long time, a strict sterile technique must
be used during the insertion and maintenance of the TPN line. The TPN line should
not be used for any other purpose. External tubing should be changed every 24 hours
with the first bag of the day. In-line filters have not been shown to decrease
complications. Dressings should be kept sterile and are usually changed every 48
hours using strict sterile techniques.

Option A: Medication therapy can continue during TPN therapy. Progress of patients
with a TPN line should be followed on a flowchart. An interdisciplinary nutrition
team, if available, should monitor patients. Weight, complete blood count,
electrolytes, and blood urea nitrogen should be monitored often (eg, daily for
inpatients). Plasma glucose should be monitored every 6 hours until patients and
glucose levels become stable. Fluid intake and output should be monitored
continuously. When patients become stable, blood tests can be done much less often.
Option C: Flushing is not required because the initiation of TPN does not require a
client to remain on bed rest during therapy. Catheter-related sepsis rates have
decreased since the introduction of guidelines that emphasize sterile techniques
for catheter insertion and skincare around the insertion site. The increasing use
of dedicated teams of physicians and nurses who specialize in various procedures
including catheter insertion also has accounted for a decrease in catheter-related
infection rates.
Option D: However, other clinical conditions of the client may affect mobility
issues and warrant the client’s being on bed rest. Place the client in a semi-
Fowler’s or high-Fowler’s position. Maintaining the head of the bed elevated will
promote ease in breathing. This position also allows the pooling of fluid in the
bases and for gas exchange to be more available to the lung tissue.
8. Question
Which of the following should be included in a plan of care for a client who is
lactose intolerant?

A. Remove all dairy products from the diet.


B. Frozen yogurt can be included in the diet.
C. Drink small amounts of milk on an empty stomach.
D. Spread out selection of dairy products throughout the day.
Incorrect
Correct Answer: B. Frozen yogurt can be included in the diet.

Clients who are lactose intolerant can digest frozen yogurt. Yogurt products are
formed by bacterial action, and this action assists in the digestion of lactose.
The freezing process further stops bacterial action so that limited lactase
activity remains. Some people who are lactose-intolerant can eat some kinds of
yogurt without problems, especially yogurt with live cultures.

Option A: Elimination of all dairy products can lead to significant clinical


deficiencies of other nutrients. Be sure to get enough calcium in the diet,
especially if the client avoids milk products completely. To get enough calcium,
the client would need to eat calcium-rich foods as often as someone would drink
milk. Calcium is very important because it keeps bones strong and reduces the risk
of osteoporosis.
Option C: Drinking milk on an empty stomach can exacerbate clinical symptoms.
Drinking milk with a meal may benefit the client because other foods, (especially
fat) may decrease transit time and allow for increased lactase activity. Limit the
amount of milk and milk products in the diet. Try to drink 1 glass of milk each
day. Drink small amounts several times a day. All types of milk contain the same
amount of lactose.
Option D: Although individual tolerance should be acknowledged, spreading out the
use of known dairy products will usually exacerbate clinical symptoms. Eat or drink
milk and milk products along with other foods. For some people, combining solid
food (like cereal) with a dairy product (like milk) can reduce symptoms.
9. Question
Pain tolerance in an elderly patient with cancer would:

A. Stay the same.


B. Be lowered.
C. Be increased.
D. No effect on pain tolerance.
Incorrect
Correct Answer: B. Be lowered.

There is potential for a lowered pain tolerance to exist with diminished adaptive
capacity. For older patients with cancer, unrelieved pain can affect functioning,
increase cognitive impairment, and depression, which in turn can influence the
severity of pain and make management more challenging. In sum, the literature
indicates that cancer pain in advanced disease is multifaceted and can adversely
affect the lives of patients and their caregivers. Changes associated with aging
have the potential to further impact this experience.

Option A: Pain continues to be a common and distressing symptom, despite pain


management being a central focus of palliative care and guidelines for the
management of cancer pain. Estimates suggest that as many as 60% to 80% of
individuals with recurrent or metastatic cancer experience pain. Among older
patients, cancer pain, similar to other types of pain, tends to go unrecognized and
undertreated
Option C: In older individuals, the higher incidence of comorbidities, age-related
declines in functioning, and associated symptoms can further complicate cancer pain
and its management. Adding to the complexity is the recognition that the experience
of pain is not merely a sensory event, but is multifaceted, comprising sensory,
affective, and evaluative components.
Option D: Indeed, the belief that pain can emanate from both physical sources and
nonphysical sources (psychological, spiritual, and interpersonal) is central to the
concept of total pain, put forth by Dame Cicely Saunders, founder of the modern
hospice. This nonphysical source of pain derives from feelings of helplessness,
being dependent on others, and having difficulty in reshaping relationships, and
has been described by terminally ill older patients as creating the worst
suffering.
10. Question
What is the main advantage of cutaneous stimulation in managing pain?

A. Costs less.
B. Restricts movement and decreases.
C. Gives client control over pain syndrome.
D. Allows the family to care for the patient at home.
Incorrect
Correct Answer: C. Gives client control over pain syndrome.

Cutaneous stimulation allows the patient to have control over his pain and allows
him to be in his own environment. Cutaneous stimulation increases movement and
decreases pain. Cutaneous stimulation involves stimulation of nerves via skin
contact in an effort to reduce pain impulses to the brain, based on the “gate
control” theory of pain. A device used to provide electrocutaneous nerve
stimulation was studied for its effect on symptoms of peripheral neuropathy.

Option A: The potential for TENS-associated improvement, combined with reduced


medication-related complications and costs, are important points that clinicians
should consider when constructing a treatment plan for chronic pain patients.
Finally, cost simulation techniques provide a useful tool for assessing outcomes in
pain treatment and research.
Option B: CS effectively reduces pain, heart rate, and blood pressure in ED
patients. The intervention of CS has solid utilization potential and could be
easily incorporated into standard ED procedures.
Option D: TENS devices can be purchased over the counter and without medical
prescription in the UK. However, a practitioner experienced in TENS principles
should supervise patients using TENS for the first time. A point of contact to
troubleshoot any problems should also be provided.
11. Question
The nurse is instructing a 65-year-old female client diagnosed with osteoporosis.
The most important instruction regarding exercise would be to

A. Exercise doing weight-bearing activities.


B. Exercise to reduce weight.
C. Avoid exercise activities that increase the risk of fracture.
D. Exercise to strengthen muscles and thereby protect bones.
Incorrect
Correct Answer: A. Exercise doing weight-bearing activities.

Weight-bearing exercises are beneficial in the treatment of osteoporosis. Although


the loss of bone cannot be substantially reversed, it can be greatly reduced if the
client includes weight-bearing exercises along with estrogen replacement and
calcium supplements in their treatment protocol. 45 minutes to one hour of aerobic
activity two to three times per week

Option B: Resistance training two or three times per week. Each session should
include exercises to strengthen the lower limb, trunk, and arm muscles, and each
exercise should be performed eight to 10 times
Option C: Balance exercises need to be at a level that is challenging to balance
and should be performed for a few minutes at least twice a week. For safety
reasons, always make sure to hold on to something if you overbalance it. People
with severe osteoporosis or kyphosis (hunching of the upper back) who are at high
risk of bone fractures may find that swimming or water exercise is their preferred
activity.
Option D: Even though walking is a weight-bearing exercise, it does not greatly
improve bone health, muscle strength, fitness or balance, unless it is carried out
at high intensity such as at a faster pace, for long durations (such as
bushwalking), or incorporates challenging terrain such as hills.
12. Question
A client in a long-term care facility complains of pain. The nurse collects data
about the client’s pain. The first step in pain assessment is for the nurse to

A. Have the client identify coping methods.


B. Get the description of the location and intensity of the pain.
C. Accept the client’s report of pain.
D. Determine the client’s status of pain.
Incorrect
Correct Answer: C. Accept the client’s report of pain.

Although all of the options above are correct, the first and most important piece
of information in this client’s pain assessment is what the client is telling you
about the pain –“the client’s report.” Pain is the most common complaint seen in a
primary care office. There are over 50 million Americans, 20 percent of all
patients, that suffer from chronic pain in the United States.

Option A: Effective treatment modalities for acute, chronic, centralized, or


neuropathic are often different. Ten percent of the United States population
complain of neuropathic pain. This population may benefit from a serotonin-
norepinephrine reuptake inhibitor (SNRI) such as duloxetine, as compared to
ibuprofen for an acute injury.
Option B: To fully assess the location of a patient’s pain, a body diagram map can
be completed. Ankle sprains are solitary, acute injuries. Body diagrams may not be
necessary in such a case. Localized pain is different from whole-body pain. Yet, in
a patient with multiple comorbid pain disorders such as fibromyalgia, centralized
pain disorder, and rheumatoid arthritis, distinguishing between the numerous
locations of a patient’s pain, as well as factoring in the radiation of their pain,
is difficult.
Option D: An essential first step in the pain assessment is distinguishing
nociceptive pain from neuropathic. Pain characterized as burning, shooting, pins,
and needles, or electric shock-like point the differential towards a neuropathic
origin of the patient’s pain Sharp or throbbing pain is more likely to be acute
nociceptive pain.
13. Question
Which statement best describes the effects of immobility in children?

A. Immobility prevents the progression of language and fine motor development.


B. Immobility in children has similar physical effects to those found in adults.
C. Children are more susceptible to the effects of immobility than are adults.
D. Children are likely to have prolonged immobility with subsequent complications.
Incorrect
Correct Answer: B. Immobility in children has similar physical effects to those
found in adults.

Care of the immobile child includes efforts to prevent complications of muscle


atrophy, contractures, skin breakdown, decreased metabolism and bone
demineralization. Secondary alterations also occur in the cardiovascular,
respiratory and renal systems. Similar effects and alterations occur in adults.

Option A: The hazards or complications of immobility, such as skin breakdown,


pressure ulcers, contractures, muscular weakness, muscular atrophy, disuse
osteoporosis, renal calculi, urinary stasis, urinary retention, urinary
incontinence, urinary tract infections, atelectasis, pneumonia, decreased
respiratory vital capacity, venous stasis, venous insufficiency, orthostatic
hypotension, decreased cardiac reserve, edema, emboli, thrombophlebitis,
constipation and the loss of calcium from the bones, are highly costly in terms of
health care dollars and in terms of client suffering.
Option C: Immobility and complete bed rest can lead to life-threatening physical
and psychological complications and consequences. Members of the nursing care team
and other health care professionals like physical therapists must, therefore,
promote client mobility and prevent immobility whenever possible. Immobility can
adversely affect all physiological bodily systems.
Option D: Children stayed in a cast for a long period, so that the effect of
postoperative immobility had negative effects on the physical and psychological
wellbeing of children with musculoskeletal injuries. Emphasize the importance of
implementing a nursing care program for children in the postoperative period for
minimizing the physical and psychological effects of immobility on children with
musculoskeletal injuries.
14. Question
After a myocardial infarction, a client is placed on a sodium-restricted diet. When
the nurse is teaching the client about the diet, which meal plan would be the most
appropriate to suggest?

A. 3 oz. broiled fish, 1 baked potato, ½ cup canned beets, 1 orange, and milk.
B. 3 oz. canned salmon, fresh broccoli, 1 biscuit, tea, and 1 apple.
C. A bologna sandwich, fresh eggplant, 2 oz fresh fruit, tea, and apple juice.
D. 3 oz. turkey, 1 fresh sweet potato, 1/2 cup fresh green beans, milk, and 1
orange.
Incorrect
Correct Answer: D. 3 oz. turkey, 1 fresh sweet potato, 1/2 cup fresh green beans,
milk, and 1 orange

Canned fish and vegetables and cured meats are high in sodium. This meal does not
contain any canned fish and/or vegetables or cured meats. Eat a Mediterranean?style
diet—more bread, fruit, vegetables, and fish; less meat; and replace butter and
cheese with products based on vegetable and plant oils (reduces total mortality and
the risk of myocardial infarction).

Option A: Consume at least 7?g of omega?3 fatty acids per week from 2–4 portions of
oily fish per week. If within 3?months of myocardial infarction and they are not
achieving this, consider providing at least 1?g daily of omega?3?acid ethyl esters
treatment licensed for secondary prevention after myocardial infarction for up to 4
years.
Option B: Choose foods with less sodium and prepare foods with little or no salt.
To lower blood pressure, aim to eat no more than 2,300 milligrams of sodium per
day. Reducing daily intake to 1,500 mg is desirable because it can lower blood
pressure even further.
Option C: Processed meats, like hot dogs, sausage, and lunch meat, are loaded with
sodium and nitrates. This can raise the blood pressure and the risk of another
heart attack. High blood pressure is particularly dangerous because there usually
aren’t any symptoms.
15. Question
A nurse is assessing several clients in a long-term health care facility. Which
client is at highest risk for the development of decubitus ulcers?

A. A 79-year-old malnourished client on bed rest.


B. An obese client who uses a wheelchair.
C. An incontinent client who has had 3 diarrhea stools.
D. An 80-year-old ambulatory diabetic client.
Incorrect
Correct Answer: A. A 79-year-old malnourished client on bed rest.

Weighing significantly less than ideal body weight increases the number and surface
area of bony prominences which are susceptible to pressure ulcers. Thus,
malnutrition is a major risk factor for decubitus, due in part to poor hydration
and inadequate protein intake. Both external and internal factors work
simultaneously, forming these ulcers. External factors; pressure, friction, shear
force, and moisture and internal factors; fever, malnutrition, anemia, and
endothelial dysfunction speed up the process of these lesions.

Option B: Immobility of as little as two hours in a bedridden patient or patient


undergoing surgery is sufficient to create the basis of a decubitus ulcer. The
dysfunction of nervous regulatory mechanisms responsible for the regulation of
local blood flow is somewhat culpable in the formation of these ulcers. Prolonged
pressure on tissues can cause capillary bed occlusion and, thus, low oxygen levels
in the area. Over time, the ischemic tissue begins to accumulate toxic metabolites.
Subsequently, tissue ulceration and necrosis occur.
Option C: The development of decubitus ulcers is complex and multifactorial. Loss
of sensory perception, locally and general impaired loss of consciousness, along
with decreased mobility, are the most important causes that aid in the formation of
these ulcers because patients are not aware of discomfort hence do not relieve the
pressure.
Option D: Elderly patients are more prone to sacral decubitus ulcers; two-thirds of
ulcers occur in patients who are over 70 years old. There is data that shows 83% of
hospitalized patients with ulcers developed them within five days of their
hospitalization.
16. Question
Mrs. Kennedy had a CVA (cerebrovascular accident) and has a severe right-sided
weakness. She has been taught to walk with a cane. The nurse is evaluating her use
of the cane prior to discharge. Which of the following reflects the correct use of
the cane?

A. Holding the cane in her left hand, Mrs. Kennedy moves the cane forward first,
then her right leg, and finally her left leg.
her left leg. B. Holding the cane in her right hand, Mrs. Kennedy moves the cane
forward first, then her left leg, and finally her right leg.
C. Holding the cane in her right hand, Mrs. Kennedy moves the cane and her right
leg forward then moves her left leg forward.
D. Holding the cane in her left hand, Mrs. Kennedy moves the cane and her left leg
forward, then moves her right leg forward.
Incorrect
Correct Answer: A. Holding the cane in her left hand, Mrs. Kennedy moves the cane
forward first, then her right leg, and finally her left leg

When a person with weakness on one side uses a cane, there should always be two
points of contact with the floor. When Mrs. Kennedy. moves the cane forward, she
has both feet on the floor, providing stability. As she moves the weak leg, the
cane and the strong leg provide support. Finally, the cane, which is even with the
weak leg, provides stability while she moves the strong leg.

Option B: She should not hold the cane with her weak arm. The use of the cane
requires arm strength to ensure that the cane provides adequate stability when
standing on the weak leg. To go upstairs, use the handrail and step up with the
unaffected leg first and follow with the cane and the affected foot together.
Option C: The cane should be held in the left hand, the hand opposite the affected
leg. Hold the cane in the hand of the unaffected side. Move the cane and the
affected leg forward at the same time, so that the cane helps take the weight of
the weak leg. Then step with the unaffected leg.
Option D: If Mrs. Kennedy. moved the cane and her strong foot at the same time, she
would be left standing on her weak leg at one point. This would be unstable at
best; at worst, impossible. To go downstairs, use the handrail and step down with
the affected foot and cane together first and follow with the unaffected foot.
17. Question
The nurse is instructing a woman on a low-fat, high-fiber diet. Which of the
following food choices, if selected by the client, indicate an understanding of a
low-fat, high-fiber diet?

A. Tuna salad sandwich on whole-wheat bread.


B. Vegetable soup made with vegetable stock, carrots, celery, and legumes served
with toasted oat bread.
C. Chef’s salad with hard-boiled eggs and fat-free dressing.
D. Broiled chicken stuffed with chopped apples and walnuts.
Incorrect
Correct Answer: B. Vegetable soup made with vegetable stock, carrots, celery, and
legumes served with toasted oat bread.

This choice shows a low-fat soup (which would have been higher in fat if made with
chicken or beef stock) and high-fiber bread and soup contents (both the vegetables
and the legumes). Eating a high-fiber diet that is low in fat can help maintain
overall health. Fiber-rich foods are naturally low in fat and contain cancer-
fighting and heart-healthy properties. While a low-fat diet is good, it is
important that the client does not dismiss all fats, however. Eat some foods
containing unsaturated fats because they are necessary for an overall healthy diet.

Option A: Mayonnaise in tuna salad is high in fat. The whole-wheat bread has some
fiber. Fiber’s presence in the digestive tract can help reduce the body’s
cholesterol absorption. This is especially true if you take statins, which are
medications to lower cholesterol, and use fiber supplements like psyllium fiber
Option C: Salad is high in fiber, but hard-boiled eggs are high in fat. In fact, a
single egg contains 212 mg of cholesterol, which is 71% of the recommended daily
intake. Plus, 62% of the calories in whole eggs are from fat.
Option D: There is some fiber in the apples and walnuts. The walnuts are high in
fat, as is the chicken. Nuts have a high-fat content, so are high in energy. In
most nuts, this is mainly unsaturated fat: either polyunsaturated fats in walnuts
and pine nuts or monounsaturated fats in almonds, pistachios, pecans, and
hazelnuts, for example. Brazil nuts, cashews, and macadamia nuts are higher in
saturated fat.
18. Question
An 85-year-old male patient has been bedridden for two weeks. Which of the
following complaints by the patient indicates to the nurse that he is developing a
complication of immobility?

A. Stiffness of the right ankle joint.


B. Soreness of the gums.
C. Short-term memory loss.
D. Decreased appetite.
Incorrect
Correct Answer: A. Stiffness of the right ankle joint.

Stiffness of a joint may indicate the beginning of contracture and/or early muscle
atrophy. In the development of joint contractures that result from long-term
immobilization, shortening of the joint capsule, synovial adhesions and
arthrofibrosis play decisive roles and may present as a generalized joint stiffness

Option B: Soreness of the gums is not related to immobility. Brushing too hard,
improper flossing techniques, infection, or gum disease can cause sore and
sensitive gums. Other causes unrelated to oral hygiene could include Vitamin K
deficiency, hormonal changes during pregnancy, leukemia, or blood disorders.
Option C: Short-term memory loss is not related to immobility. Short-term memory
loss is when one forgets things they heard, saw, or did recently. It’s a normal
part of getting older for many people. But it can also be a sign of a deeper
problem, such as dementia, a brain injury, or a mental health issue.
Option D: Decreased appetite is unlikely to be related to immobility. People can
experience a loss of appetite for a wide range of reasons. Some of these are short-
term, including colds, food poisoning, other infections, or the side effects of
medication. Others are to do with long-term medical conditions, such as diabetes,
cancer, or life-limiting illnesses.
19. Question
An eleven-month-old infant is brought to the pediatric clinic. The nurse suspects
that the child has iron-deficiency anemia. Because iron deficiency anemia is
suspected, which of the following is the most important information to obtain from
the infant’s parents?

A. Normal dietary intake.


B. Relevant socio-cultural, economic, and educational background of the family.
C. Any evidence of blood in the stools.
D. A history of maternal anemia during pregnancy.
Incorrect
Correct Answer: A. Normal dietary intake.

Iron deficiency anemia occurs commonly in children 6 to 24 months of age. For the
first 4 to 5 months of infancy iron stores laid down for the baby during pregnancy
are adequate. When fetal iron stores are depleted, supplemental dietary iron needs
to be supplied to meet the infant’s rapid growth needs. Iron deficiency may occur
in the infant who drinks mostly milk, which contains no iron, and does not receive
adequate dietary iron or supplemental iron.

Option B: Daily dietary intake is much more related to the diagnosis of iron
deficiency anemia than is the socio-cultural, economic, and educational background
of the family. The cause of iron-deficiency anemia varies based on age, gender, and
socioeconomic status. Iron deficiency may result from insufficient iron intake,
decreased absorption, or blood loss.
Option C: Iron deficiency anemia in an infant is very unlikely to be related to
gastrointestinal bleeding. In developing countries, a parasitic infestation is also
a significant cause of iron deficiency anemia. Dietary sources of iron are green
vegetables, red meat, and iron-fortified milk formulas.
Option D: Anemia during pregnancy is unlikely to be the cause of the infant’s iron
deficiency anemia. Fetal iron stores are drawn from the mother even if she is
anemic. In neonates, breastfeeding is protective against iron deficiency due to the
higher bioavailability of iron in breast milk compared to cow’s milk; iron
deficiency anemia is the most common form of anemia in young children on cow’s
milk.
20. Question
A 46-year-old female with chronic constipation is assessed by the nurse for a bowel
training regimen. Which factor indicates further information is needed by the
nurse?

A. The client’s dietary habits include foods high in bulk.


B. The client’s fluid intake is between 2500-3000 ml per day.
C. The client engages in moderate exercise each day.
D. The client’s bowel habits were not discussed.
Incorrect
Correct Answer: D. The client’s bowel habits were not discussed.

To assess the client for a bowel training program the factors causing the bowel
alteration should be assessed. A routine for bowel elimination should be based on
the client’s previous bowel habits and alterations in bowel habits that have
occurred because of illness or trauma.

Option A: Foods high in bulk are appropriate. Assist the patient to take at least
20 g of dietary fiber (e.g., raw fruits, fresh vegetables, whole grains) per day.
Fiber adds bulk to the stool and makes defecation easier because it passes through
the intestine essentially unchanged.
Option B: The client and the family should assist in the planning of the program
which should include foods high in bulk, adequate exercise, and fluid intake of
2500-3000 ml. Encourage the patient to take in fluid 2000 to 3000 mL/day, if not
contraindicated medically. Sufficient fluid is needed to keep the fecal mass soft.
But take note of some patients or older patients having cardiovascular limitations
requiring less fluid intake.
Option C: Exercise should be a part of a bowel training regimen. Urge the patient
for some physical activity and exercise. Consider isometric abdominal and gluteal
exercises. Movement promotes peristalsis. Abdominal exercises strengthen abdominal
muscles that facilitate defecation.
21. Question
Mr. Teban is a 73-year old patient diagnosed with pneumonia. Which data would be of
greatest concern to the nurse when completing the nursing assessment of the
patient?

A. Alert and oriented to date, time, and place.


B. Buccal cyanosis and capillary refill greater than 3 seconds.
C. Clear breath sounds and nonproductive cough.
D. Hemoglobin concentration of 13 g/dl and leukocyte count 5,300/mm3.
Incorrect
Correct Answer: B. Buccal cyanosis and capillary refill greater than 3 seconds.

Buccal cyanosis and capillary refill greater than 3 seconds are indicative of
decreased oxygen to the tissues, which requires immediate intervention. As
oxygenation and perfusion become impaired, peripheral tissues become cyanotic.
Cyanosis of nail beds may represent vasoconstriction or the body’s response to
fever/chills; however, cyanosis of earlobes, mucous membranes, and skin around the
mouth (“warm membranes”) is indicative of systemic hypoxemia. Alert and oriented,
clear breath sounds, nonproductive cough, hemoglobin concentration of 13 g/dl, and
leukocyte count of 5,300/mm3 are normal data.

Option A: Restlessness, irritation, confusion, and somnolence may reflect hypoxemia


and decreased cerebral oxygenation and may require further intervention. Check
pulse oximetry results with any mental status changes in older adults.
Option C: Decreased airflow occurs in areas with consolidated fluid. Bronchial
breath sounds can also occur in these consolidated areas. Crackles, rhonchi, and
wheezes are heard on inspiration and/or expiration in response to fluid
accumulation, thick secretions, and airway spasms and obstruction.
Option D: Monitor serial chest x-rays, ABGs, pulse oximetry readings. Follows
progress and effects and extent of pneumonia. Therapeutic regimen, and may
facilitate necessary alterations in therapy. Oxygen saturation should be maintained
at 90% or greater. Imbalances in PaCO2 and PaO2 may indicate respiratory fatigue.
22. Question
During the nursing assessment, which data represent information concerning health
beliefs?

A. Family role and relationship patterns.


B. Educational level and financial status.
C. Promotive, preventive, and restorative health practices.
D. Use of prescribed and over-the-counter medications.
Incorrect
Correct Answer: C. Promotive, preventive, and restorative health practices.

The health-beliefs assessment includes expectations of health care; promotive,


preventive, and restorative practices, such as breast self-examination, testicular
examination, and seat-belt use; and how the client perceives illness. The basic
premise of the health belief assessment is that patients have a right to their
cultural beliefs, values, and practices, and that these factors should be
understood, respected, and considered when giving culturally competent care.

Option A: Educational level and financial status represent information associated


with role and relationship patterns. Critical thinking skills applied during the
nursing process provide a decision-making framework to develop and guide a plan of
care for the patient incorporating evidence-based practice concepts. This concept
of precision education to tailor care based on an individual’s unique cultural,
spiritual, and physical needs, rather than a trial by error, one size fits all
approach results in a more favorable outcome.
Option B: Family role and relationship patterns represent information associated
with role and relationship patterns. The nursing assessment includes gathering
information concerning the patient’s individual physiological, psychological,
sociological, and spiritual needs. It is the first step in the successful
evaluation of a patient. Subjective and objective data collection are an integral
part of this process.
Option D: The use of medications provides information about the client’s personal
habits. Part of the assessment includes data collection by obtaining vital signs
such as temperature, respiratory rate, heart rate, blood pressure, and pain level
using age or condition-appropriate pain scale. The assessment identifies the
current and future care needs of the patient by allowing the formation of a nursing
diagnosis. The nurse recognizes normal and abnormal patient physiology and helps
prioritize interventions and care.
23. Question
Nurse Patrick is acquiring information from a client in the emergency department.
Which is an example of biographic information that may be obtained during a health
history?

A. The chief complaint


B. Past health status
C. History immunizations
D. Location of an advance directive
Incorrect
Correct Answer: D. Location of an advance directive

Biographic information may include name, address, gender, race, occupation, and
location of a living will or durable power of attorney for health care. Biographic
data usually include information that identifies the client, such as name, address,
phone number, gender, and who provided the information—the client or significant
others. The client’s birth date, Social Security number, medical record number, or
similar identifying data may be included in the biographic data section. The chief
complaint, past health status, and history of immunizations are part of assessing
the client’s health and illness patterns.

Option A: Encourage the client to explain the health problem or symptom in as much
detail as possible by focusing on the onset, progression, and duration of the
problem; signs and symptoms and related problems; and what the client perceives as
causing the problem. The client’s answers to the questions provide the nurse with a
great deal of information about the client’s problem and especially how it affects
the lifestyle and activities of daily living. This helps the nurse to evaluate the
client’s insight into the problem and the client’s plans for managing it.
Option B: This portion of the health history focuses on questions related to the
client’s past, from the earliest beginnings to the present. These questions elicit
data related to the client’s strengths and weaknesses in her health history. The
client’s strengths may be physical (e.g., optimal body weight), social (e.g.,
active in community services), emotional (e.g., expresses feeling openly), or
spiritual (often turns to faith for support).
Option C: Information covered in the past health history includes questions about
birth, growth, development, childhood diseases, immunizations, allergies, previous
health problems, hospitalizations, surgeries, pregnancies, births, previous
accidents, injuries, painful experiences, and emotional or psychiatric problems.
24. Question
John Joseph was scheduled for a physical assessment. When percussing the client’s
chest, the nurse would expect to find which assessment data as a normal sign over
his lungs?

A. Dullness
B. Resonance
C. Hyperresonance
D. Tympany
Incorrect
Correct Answer: B. Resonance

Normally, when percussing a client’s chest, percussion over the lungs reveals
resonance, a hollow or loud, low-pitched sound of long duration. Since lungs are
mostly filled with air that we breathe in, percussion performed over most of the
lung area produces a resonant sound, which is a low-pitched, hollow sound.
Therefore, any dullness or hyper-resonance is indicative of lung pathology, such as
pleural effusion or pneumothorax, respectively.

Option A: Dullness is typically heard on percussion of solid organs, such as the


liver or areas of consolidation. Dullness to percussion indicates denser tissue,
such as zones of effusion or consolidation. Once an abnormality is detected,
percussion can be used around the area of interest to define the extent of the
abnormality. Normal areas of dullness are those overlying the liver and spleen at
the anterior bases of the lungs.
Option C: Hyperresonance would be evidenced by percussion over areas of
overinflation such as an emphysematous lung. Hyperresonant sounds may also be heard
when percussing lungs hyperinflated with air, such as may occur in patients with
COPD, or patients having an acute asthmatic attack. An area of hyper resonance on
one side of the chest may indicate a pneumothorax.
Option D: Tympany is typically heard on percussion over such areas as a gastric air
bubble or the intestine. Tympanic sounds are hollow, high, drumlike sounds. Tympany
is normally heard over the stomach but is not a normal chest sound. Tympanic sounds
heard over the chest indicate excessive air in the chest, such as may occur with
pneumothorax.
25. Question
Matteo is diagnosed with dehydration and underwent a series of tests. Which
laboratory result would warrant immediate intervention by the nurse?

A. Serum sodium level of 138 mEq/L


B. Serum potassium level of 3.1 mEq/L
C. Serum glucose level of 120 mg/dl
D. Serum creatinine level of 0.6 mg/100 ml
Incorrect
Correct Answer: B. Serum potassium level of 3.1 mEq/L

A normal potassium level is 3.5 to 5.5 mEq/L. Hypokalemia is more prevalent than
hyperkalemia, and most cases are mild. Severity is categorized as mild when the
serum potassium level is 3 to 3.4 mmol/L, moderate when the serum potassium level
is 2.5 to 3 mmol/L, and severe when the serum potassium level is less than 2.5
mmol/L.

Option A: A normal sodium level is 135 to 145 mEq/L. Among the electrolyte
disorders, hyponatremia is the most frequent. Diagnosis is when the serum sodium
level is less than 135 mmol/L. Hyponatremia has neurological manifestations.
Patients may present with headache, confusion, nausea, delirium. Hypernatremia
presents when the serum sodium levels are greater than145 mmol/L.
Option C: A normal non fasting glucose level is 85 to 140 mg/dl. Normal plasma
glucose levels are defined as under 100 mg/dL during fasting and less than 140
mg/dL 2-hours postprandial. Additionally, glucose levels in healthy individuals can
vary with age. Fasting plasma glucose in adults tends to increase with age starting
in the third decade of life but does not increase significantly beyond 60 years of
age. Normal HbA1c is lower than 5.7%.
Option D: A normal creatinine level is 0.2 to 0.8 mg/100 ml. Serum creatinine level
for men with normal kidney function is approximately 0.6 to 1.2mg/dL and between
0.5 to 1.1 mg/dL for women. Alteration of serum creatinine values can occur as its
generation is subject to influence by muscle function, activity, diet, and health
status of the patient. Increased tubular secretion of creatinine in certain
patients with dysfunctional kidneys could provide a false negative value. Elevated
serum creatinine levels are also present in patients with muscular dystrophy
paralysis, anemia, leukemia, and hyperthyroidism.
26. Question
During an otoscopic examination, which action should be avoided to prevent the
client from discomfort and injury?

A. Tipping the client's head away from the examiner and pulling the ear up and
back.
B. Inserting the otoscope inferiorly into the distal portion of the external
canal.
C. Inserting the otoscope superiorly into the proximal two-thirds of the external
canal.
D. Bracing the examiner's hand against the client's head.
Incorrect
Correct Answer: C. Inserting the otoscope superiorly into the proximal two-thirds
of the external canal.

In the superior position, the speculum of the otoscope is nearest the tympanic
membrane, and the most sensitive portion of the external canal is the proximal two-
thirds. It is important to avoid these structures during the examination. The
provider should then slowly progress the speculum into the canal until the tympanic
membrane becomes visible. The provider should evaluate the health of the tympanic
membrane and observe factors such as color, presence of perforation, and a bulging
appearance.

Option A: With the hand that is not holding the otoscope, the provider should grasp
and gently pull the patient’s pinna to help straighten the patient’s external
auditory canal. This step will facilitate visualization of the tympanic membrane.
In a child, the examiner should pull the pinna posteriorly and inferiorly. In an
adult, the examiner should pull the pinna posteriorly and superiorly.
Option B: During the otoscopic examination, the provider utilizes an otoscope, also
known as an auriscope, to visualize the ear anatomy. While performing the otoscopic
examination, the provider holds the handle of the otoscope and inserts the cone of
the otoscope into the patient’s external auditory canal.
Option D: Providers may have their own preferences regarding how to grasp the
otoscope. However, it is generally advisable to hold the otoscope like a pen in
between the first and second fingers. The otoscope is usually held in the right
hand when evaluating the patient’s right ear and the left hand when assessing the
patient’s left ear. The provider should place their free fifth finger of the hand,
holding the otoscope against the patient’s cheek to support and brace the hand
during the examination.
27. Question
When assessing the lower extremities for arterial function, which intervention
should the nurse perform?

A. Assessing the medial malleoli for pitting edema.


B. Performing Allen's test.
C. Assessing the Homans' sign.
D. Palpating the pedal pulses.
Incorrect
Correct Answer: D. Palpating the pedal pulses.

Palpating the client’s pedal pulses assists in determining if arterial blood supply
to the lower extremities is sufficient. Finding a pedal pulse is part of the trauma
patient assessment and performed before and after lower extremity splint
application as well as long backboard immobilization. Assessing a pedal pulse is
part of the ongoing assessment for a patient on a backboard or a lower extremity
splint.

Option A: Assessing the medial malleoli for pitting edema is appropriate for
assessing the venous function of the lower extremity. The lower extremity
examination should focus on the medial malleolus, the bony portion of the tibia,
and the dorsum of the foot. Pitting edema also occurs in the early stages of
lymphedema because of an influx of protein-rich fluid into the interstitium, before
fibrosis of the subcutaneous tissue; therefore, its presence should not exclude the
diagnosis of lymphedema.
Option B: Allen’s test is used to evaluate arterial blood flow before inserting an
arterial line in an upper extremity or obtaining arterial blood gases. The Allen
test is a first-line standard test used to assess the arterial blood supply of the
hand. This test is performed whenever intravascular access to the radial artery is
planned or for selecting patients for radial artery harvesting, such as for
coronary artery bypass grafting or for forearm flap elevation.
Option C: Homans’ sign is used to evaluate the possibility of deep vein thrombosis.
Homan’s sign test also called dorsiflexion sign test is a physical examination
procedure that is used to test for Deep Vein Thrombosis (DVT). A positive Homan’s A
positive Homan’s sign in the presence of other clinical signs may be a quick
indicator of DVT.
28. Question
Newly hired nurse Liza is excited to perform her very first physical assessment
with a 19-year-old client. Which assessment examination requires Liza to wear
gloves?

A. Breast
B. Integumentary
C. Ophthalmic
D. Oral
Incorrect
Correct Answer: D. Oral

Gloves should be worn anytime there is a risk of exposure to the client’s blood or
body fluids. Oral, rectal, and genital examinations require gloves because they
involve contact with body fluids. Ophthalmic, breast, or integumentary examinations
normally do not involve contact with the client’s body fluids and do not require
the nurse to wear gloves for protection.

Option A: After completing the visual inspection, the patient should be instructed
to lay supine. If a site-specific breast complaint is being evaluated, the examiner
should begin his/her exam on the opposite, or “normal” side. As one breast is
examined, the other is covered for the patient’s comfort. The patient should place
the ipsilateral hand above and/or behind their head to flatten the breast tissue as
much as possible. The breast tissue itself is evaluated using a sequence of
palpation that allows serial progression from superficial to deeper tissues.
Option B: A general assessment of the skin begins at the initial contact with the
patient and continues throughout the examination. Specific areas of the skin are
assessed during the examination of other body systems unless the chief complaint is
a dermatologic problem. However, if there are areas of skin breakdown or drainage,
gloves should be used.
Option C: The Royal College of Ophthalmologists have updated their advice on PPE to
ophthalmologists and are now recommending that clinicians should wear standard
surgical masks when examining or treating patients at the slit lamp. Gowns and
gloves are not recommended. They also recommend that plastic breath shields
attached to slit lamps provide some protection, but must be disinfected between
patients as studies show that the COVID-19 virus is viable for up to 72 hours on
plastic surfaces.
29. Question
Nurse Renner is about to perform Romberg’s test on Pierro. To ensure the latter’s
safety, which intervention should nurse Renner implement?

A. Allowing the client to keep his eyes open.


B. Having the client hold on to furniture.
C. Letting the client spread his feet apart.
D. Standing close to provide support.
Incorrect
Correct Answer: D. Standing close to provide support.

During Romberg’s test, the client is asked to stand with feet together and eyes
shut and still maintain balance with the minimum of sway. If the client loses his
balance, the nurse standing close to provide support, such as having an arm close
around his shoulder, can prevent a fall. Allowing the client to keep his eyes open,
spread his feet apart, or hang on to a piece of furniture interferes with the
proper execution of the test and yields invalid results.

Option A: The clinician asks the patient to first stand quietly with eyes open, and
subsequently with eyes closed. The patient tries to maintain his balance. For
safety, it is essential that the observer stand close to the patient to prevent
potential injury if the patient were to fall. When the patient closes his eyes, he
should not orient himself by light, sense or sound, as this could influence the
test result and cause a false positive outcome.
Option B: In the Romberg test, the patient stands upright and asked to close his
eyes. A loss of balance is interpreted as a positive Romberg sign. The Romberg test
is positive when the patient is unable to maintain balance with their eyes closed.
Losing balance can be defined as increased body sway, placing one foot in the
direction of the fall, or even falling.
Option C: The patient is asked to remove his shoes and stand with his two feet
together. The arms are held next to the body or crossed in front of the body. If
the clinician observes that the patient is able to stand for long periods of time
with the eyes closed, it is evident that the patient’s balance and proprioceptive
deficits have decreased.
30. Question
A physical assessment is being performed on patient Geoff by Nurse Tine. During the
abdominal examination, Nurse Tine should perform the four physical examination
techniques in which sequence?

A. Auscultation immediately after the inspection and then percussion and


palpation.
B. Percussion, followed by inspection, auscultation, and palpation.
C. Palpation of tender areas first and then inspection, percussion, and
auscultation.
D. Inspection and then palpation, percussion, and auscultation.
Incorrect
Correct Answer: A. Auscultation immediately after the inspection and then
percussion and palpation

With an abdominal assessment, auscultation always is performed before percussion


and palpation because any abdominal manipulation, such as from palpation or
percussion, can alter bowel sounds. Assessing the patient’s abdomen can provide
critical information about his internal organs. Always follow this sequence:
inspection, auscultation, percussion, and palpation. Changing the order of these
assessment techniques could alter the frequency of bowel sounds and make the
findings less accurate.

Option B: Percussion should never precede inspection or auscultation, and any


tender or painful areas should be palpated last. Assess for any visible mass,
bulging, or asymmetry. Look for unusual coloring, scars, striae, lesions,
petechiae, ecchymoses, spider angiomas, and suspicious-looking moles. Inspect the
umbilicus and note any hernias. Look for pulsations. A thin patient may have a
pulsation of the aorta in his epigastric area and possibly peristaltic waves.
Option C: Lightly percuss all four quadrants of the patient’s abdomen. You’ll hear
dull sounds over solid structures (such as the liver) and fluid-filled structures
(such as a full bladder). Air-filled areas (such as the stomach) produce tympany.
Dullness is a normal finding over the liver, but a large, dull area elsewhere may
indicate a tumor or mass.
Option D: Using a light, gentle, dipping motion, palpate for abnormalities, such as
muscle guarding, rigidity, or superficial masses. Palpate clockwise, lifting
fingers as you move from one location to another. After light palpation of the
entire abdomen, place a non-dominant hand on the dominant hand to perform deeper
palpation (1½ to 2 inches [3.8 to 5 cm]). However, avoid deep palpation if the
patient may have a problem such as splenomegaly, appendicitis, or aneurysm or if
palpation is painful for any reason.
31. Question
Which assessment data should the nurse include when obtaining a review of body
systems?

A. Brief statement about what brought the client to the health care provider.
B. Client complaints of chest pain, dyspnea, or abdominal pain.
C. Information about the client's sexual performance and preference.
D. The client's name, address, age, and phone number.
Incorrect
Correct Answer: B. Client complaints of chest pain, dyspnea, or abdominal pain.

Client complaints about chest pain, dyspnea, or abdominal pain are considered part
of the review of body systems. This portion of the assessment elicits subjective
information on the client’s perceptions of major body system functions, including
cardiac, respiratory, and abdominal. Critical thinking skills applied during the
nursing process provide a decision-making framework to develop and guide a plan of
care for the patient incorporating evidence-based practice concepts.

Option A: A brief statement about what brought the client to the health care
provider is the chief complaint. The CC is the reason for the visit as stated in
the patient’s own words. This must be present for each encounter, and should
reference a specific condition or complaint (e.g., patient complains of abdominal
pain).
Option C: Information about the client’s sexual performance and preference
addresses past health status. Understanding the client’s current and past health is
important and may provide an explanation or rationale for the client’s current
health status. Furthermore, these data can provide insight into health promotion
needs and co-morbidities. It is helpful to understand the current and past health
profiles before assessing other aspects of health, as the information will inform
subsequent questions.
Option D: The client’s name, address, age, and phone number are biographical data.
“Introductory Information” refers to the demographic and biographic data that you
collect from the client. This data provides you with basic characteristics about
the client, such as their name, contact information, birth date and age, gender and
preferred pronouns, allergies, languages spoken and preferred language,
relationship status, occupation, and resuscitation status.
32. Question
Tywin has come to the nursing clinic for a comprehensive health assessment. Which
statement would be the best way to end the history interview?

A. "What brought you to the clinic today?"


B. "Would you describe your overall health as good?"
C. "Do you understand what is happening?"
D. "Is there anything else you would like to tell me?"
Incorrect
Correct Answer: D. “Is there anything else you would like to tell me?”

By asking the client if there is anything else, the nurse allows the client to end
the interview by discussing feelings and concerns. The purpose of obtaining a
health history is to gather subjective data from the patient and/or the patient’s
family so that the health care team and the patient can collaboratively create a
plan that will promote health, address acute health problems, and minimize chronic
health conditions.

Option A: Asking about what brought the client to the clinic is an ambiguous
question to which the client may answer “my car” or any similarly disingenuous
reply. The health history is typically done on admission to the hospital, but a
health history may be taken whenever additional subjective information from the
patient may be helpful to inform care (Wilson & Giddens, 2013).
Option B: Asking if the client describes his overall health as good is a leading
question that puts words in his mouth. Objective data is information that the
health care professional gathers during a physical examination and consists of
information that can be seen, felt, smelled, or heard by the health care
professional. Taken together, the data collected provides a health history that
gives the health care professional an opportunity to assess health promotion
practices and offer patient education (Stephen et al., 2012).
Option C: Asking if the client understands what is happening is a yes-or-no
question that can elicit little information. Data gathered may be subjective or
objective in nature. Subjective data is information reported by the patient and may
include signs and symptoms described by the patient but not noticeable to others.
Subjective data also includes demographic information, patient and family
information about past and current medical conditions, and patient information
about surgical procedures and social history.
33. Question
For which time period would the nurse notify the health care provider that the
client had no bowel sounds?

A. 2 minutes
B. 3 minutes
C. 4 minutes
D. 5 minutes
Incorrect
Correct Answer: D. 5 minutes

To completely determine that bowel sounds are absent, the nurse must auscultate
each of the four quadrants for at least 5 minutes; 2, 3, or 4 minutes is too short
a period to arrive at this conclusion. The first item to listen for is the presence
of bowel sounds. To chart an assessment finding of no bowel sounds, the nurse needs
to listen over the quadrant for at least five minutes. The nurse should also do the
auscultation before palpation and percussion to avoid influencing bowel sounds.

Option A: In most cases, bowel sounds are present, but the nurse needs to
categorize them. She should listen for the intensity of the sound – whether it is
soft or strong. The nurse should also listen for frequency. Hypoactive bowel sounds
could indicate a problem, so if the nurse is having trouble hearing them, this is
significant.
Option B: Auscultating bowel sounds can allow the nurse to pinpoint areas where an
obstruction may have occurred. Finding no bowel sounds can mean an ileus or
obstruction above that area of the intestine.
Option C: Hypoactive bowel sounds are considered as one every three to five
minutes, and this can indicate diarrhea, anxiety, or gastroenteritis. Hyperactive
bowel sounds are often found before a blockage. It is quite common to find one
quadrant with hyperactive bowel sounds and one with none or hypoactive ones.
34. Question
Evaluating the apical pulse is the most reliable noninvasive way to assess cardiac
function. Which is the best area for auscultating the apical pulse?

A. Aortic arch
B. Pulmonic area
C. Tricuspid area
D. Mitral area
Incorrect
Correct Answer: D. Mitral area

The mitral area (also known as the left ventricular area or the apical area), the
fifth intercostal space (ICS) at the left midclavicular line, is the best area for
auscultating the apical pulse. The apical pulse is auscultated with a stethoscope
over the chest where the heart’s mitral valve is best heard. In infants and young
children, the apical pulse is located at the fourth intercostal space at the left
midclavicular line. In adults, the apical pulse is located at the fifth intercostal
space at the left midclavicular line.

Option A: The aortic arch is the second ICS to the right of the sternum. Apical
pulse rate is indicated during some assessments, such as when conducting a
cardiovascular assessment and when a client is taking certain cardiac medications
(e.g., digoxin). Sometimes the apical pulse is auscultated pre and post medication
administration.
Option B: The pulmonic area is the second intercostal space to the left of the
sternum. It is also a best practice to assess apical pulse in infants and children
up to five years of age because radial pulses are difficult to palpate and count in
this population. It is typical to assess apical pulses in children younger than
eighteen, particularly in hospital environments. Apical pulses may also be taken in
obese people because their peripheral pulses are sometimes difficult to palpate.
Option C: The tricuspid area is the fifth ICS to the left of the sternum. Position
the client in a supine (lying flat) or in a seated position. Physically palpate the
intercostal spaces to locate the landmark of the apical pulse. Ask the female
client to re-position her own breast tissue to auscultate the apical pulse.
35. Question
Beginning in their 20s, women should be told about the benefits and limitations of
breast self-exam (BSE). Which scientific rationale should the nurse remember when
performing a breast examination on a female client?

A. One-half of all breast cancer deaths occurs in women ages 35 to 45.


B. The tail of Spence area must be included in the self-examination.
C. The position of choice for the breast examination is supine.
D. A pad should be placed under the opposite scapula of the breast being palpated.
Incorrect
Correct Answer: B. The tail of Spence area must be included in the self-
examination.

The tail of Spence, an extension of the upper outer quadrant of breast tissue, can
develop breast tumors. This area must also be included in breast self-examination.
As the fingers traverse the breasts, they must remain in contact with the skin to
avoid missing any tissue plane. Assessment of the inner half of the breasts
requires changing to a supine position, removing the hand from the forehead, and
placing the inactive arm at a right angle on the examination surface.

Option A: One-half of all women who die of breast cancer are older than age 65.
Breast cancer is the most prevalent malignancy among female populations and is
responsible for the second-highest number of cancer-related deaths in American
women. The need for early detection has manifested several screening initiatives
intent on curtailing morbidity and mortality associated with the disease.
Option C: The correct position for breast self-examination is not limited to the
supine position; the sitting position with hands at sides, above head, and on the
hips is also recommended. A visual survey of the breast tissue requires an
inspection from three angles, with arms at the side, arms raised above the head
while bending forward, and hunched over with the hands placed on the hips. Each of
these positions should be observed in a mirror from a direct view, right profile,
and left profile.
Option D: A pad is placed under the ipsilateral (e.g., same side) scapula of the
breast being palpated. The middle fingertip pad should be used to perform small
circles with light, medium, and deep pressure investigating varying depths of
breast tissue. To complete the examination of the breast’s outer half, up and down
motions of palpation are performed medially from the axilla to the nipple and
vertically from the clavicle to just below the bra line.
36. Question
Mr. Lim, who has chronic pain, loss of self-esteem, no job, and bodily
disfigurement from severe burns over the trunk and arms, is admitted to a pain
center. Which evaluation criteria would indicate the client’s successful
rehabilitation?

A. The client remains free of the aftermath phase of the pain experience.
B. The client experiences decreased frequency of acute pain episodes.
C. The client continues normal growth and development with intact support systems.
D. The client develops increased tolerance for severe pain in the future.
Incorrect
Correct Answer: C. The client continues normal growth and development with intact
support systems.

Even though the client may experience an aftermath phase, progress is still
possible, as is effective rehabilitation. Give positive reinforcement of progress
and encourage endeavors toward the attainment of rehabilitation goals. Words of
encouragement can support the development of positive coping behaviors.

Option A: Aftermath reactions may occur but need not interfere with rehabilitation.
Encourage family interaction with each other and with the rehabilitation team. To
open lines of communication and provide ongoing support for the patient and family.
Option B: Acute pain is not expected at this stage of recovery. Pain is nearly
always present to some degree because of varying severity of tissue involvement and
destruction but is usually most severe during dressing changes and debridement.
Option D: Conditioning probably would produce less pain tolerance. Exercise is
generally considered to be a safe and efficacious approach to restoring
physiological function in patients with various chronic diseases. However, the
inclusion of exercise regimens in the outpatient rehabilitation of patients who
have undergone major trauma, such as a large burn, is not common.
37. Question
Christine Ann is about to take her NCLEX examination next week and is currently
reviewing the concept of pain. Which scientific rationale would indicate that she
understands the topic?

A. Pain is an objective sign of a more serious problem.


B. Pain sensation is affected by a client's anticipation of pain.
C. Intractable pain may be relieved by treatment.
D. Psychological factors rarely contribute to a client's pain perception.
Incorrect
Correct Answer: B. Pain sensation is affected by a client’s anticipation of pain.

Phases of pain experience include the anticipation of pain. Fear and anxiety affect
a person’s response to sensation and typically intensify the pain. Similarly, other
factors such as cognitive appraisal of the meaning of the sensation, the emotional
and psychophysiological reactions, expectations, and coping skills can all serve as
feedback to influence pain perception.

Option A: Pain is a subjective sensation that cannot be quantified by anyone except


the person experiencing it. Pain refers to the product of higher brain center
processing; it entails the actual unpleasant emotional and sensory experience
generated from nervous signals. Reports of pain are thus not merely a direct output
of nociception, they involve interaction with numerous inputs (attention, affective
dimensions, autonomic variables, immune variables, and more), and may be considered
more accurately from the perspective of a neuromatrix.
Option C: Intractable pain is moderate to severe pain that cannot be relieved by
any known treatment. Intractable pain refers to a type of pain that can’t be
controlled with standard medical care. Intractable essentially means difficult to
treat or manage. This type of pain isn’t curable, so the focus of treatment is to
reduce the discomfort.
Option D: Psychological factors contribute to a client’s pain perception. In many
cases, pain results from emotions, such as hostility, guilt, or depression. There
are a number of psychological processes behind pain perception. Attentional
orienting to the painful sensation and its source can serve to heighten the painful
experience. For instance, patients with somatic preoccupation and hypochondriasis
are found to over-attend to bodily sensations, amplifying them as pain.
38. Question
Miggy, a 6-year-old boy, received a small paper cut on his finger, his mother let
him wash it and apply a small amount of antibacterial ointment and bandage. Then
she let him watch TV and eat an apple. This is an example of which type of pain
intervention?

A. Pharmacologic therapy
B. Environmental alteration
C. Control and distraction
D. Cutaneous stimulation
Incorrect
Correct Answer: C. Control and distraction

The mother’s actions are an example of control and distraction. Involving the child
in care and providing distraction took his mind off the pain. The brain can only
focus its attention in so many areas at one time. Pain sensations compete for
attention with all of the other things going on around. Just how much attention the
brain gives each thing depends on a number of factors, including how long you have
been hurting and the current mood.

Option A: Pharmacologic agents for pain analgesics — were not used. A wide range of
drugs are used to manage pain resulting from inflammation in response to tissue
damage, chemical agents/pathogens (nociceptive pain) or nerve damage (neuropathic
pain).
Option B: The home environment was not changed. There has recently been heightened
recognition that environmental factors can influence pain. Clinicians involved in
delivering multidisciplinary pain programs often structure the social environment
of their treatment settings to help promote adaptive responses to pain.
Option D: Cutaneous stimulation, such as massage, vibration, or pressure, was not
used. Cutaneous stimulation involves stimulation of nerves via skin contact in an
effort to reduce pain impulses to the brain, based on the “gate control” theory of
pain. A device used to provide electrocutaneous nerve stimulation was studied for
its effect on symptoms of peripheral neuropathy.
39. Question
Which statement represents the best rationale for using noninvasive and non-
pharmacologic pain-control measures in conjunction with other measures?

A. These measures are more effective than analgesics.


B. These measures decrease input to large fibers.
C. These measures potentiate the effects of analgesics.
D. These measures block transmission of type C fiber impulses.
Incorrect
Correct Answer: C. These measures potentiate the effects of analgesics.

Noninvasive measures may result in the release of endogenous molecular


neuropeptides with analgesic properties. They potentiate the effect of analgesics.
The role of non-pharmacological approaches to pain management is evolving, and some
non-pharmacological and complementary therapies have an increasingly important
contribution to make to holistic patient care alongside analgesics.

Option A: No evidence indicates that noninvasive and nonpharmacologic measures are


more effective than analgesics in relieving pain. Exercise, multidisciplinary
rehabilitation, acupuncture, CBT, mindfulness practices, massage, and mind-body
practices most consistently improve function and/or pain beyond the course of
therapy for specific chronic pain conditions.
Option B: Decreased input over large fibers allows more pain impulses to reach the
central nervous system. When deciding the most effective non-pharmacological
technique, take into consideration the patient’s age, developmental level, medical
history and prior experiences, the current degree of pain, and/or anticipated pain.
The advantage of non-pharmacological treatments is that they are relatively
inexpensive and safe.
Option D: There is no connection between type C fiber impulses and noninvasive and
nonpharmacologic pain-control measures. Non-pharmacological pain therapy refers to
interventions that do not involve the use of medications to treat pain. The goals
of non-pharmacological interventions are to decrease fear, distress, and anxiety,
and reduce pain and provide patients with a sense of control.
40. Question
When evaluating a client’s adaptation to pain, which behavior indicates appropriate
adaptation?

A. The client distracts himself during pain episodes.


B. The client denies the existence of any pain.
C. The client reports no need for family support.
D. The client reports pain reduction with decreased activity.
Incorrect
Correct Answer: A. The client distracts himself during pain episodes.

Distraction is an appropriate method of reducing pain. This technique involves


heightening one’s concentration upon non-painful stimuli to decrease one’s
awareness and experience of pain. Drawing the person away from the pain lessens the
perception of pain. Examples include reading, watching TV, playing video games, and
guided imagery.

Option B: Denying the existence of any pain is inappropriate and not indicative of
coping. It is essential to assist patients to express as factually as possible
(i.e., without the effect of mood, emotion, or anxiety) the effect of pain relief
measures. Inconsistencies between behavior or appearance and what the patient says
about pain relief (or lack of it) may be more a reflection of other methods the
patient is using to cope with the pain rather than pain relief itself.
Option C: Exclusion of family members and other sources of support represents a
maladaptive response. Nurses have the duty to ask their clients about their pain
and believe their reports of pain. Challenging or undermining their pain reports
results in an unhealthy therapeutic relationship that may hinder pain management
and deteriorate rapport.
Option D: Range-of-motion exercises and at least mild activity, not a decreased
activity, can help reduce pain and are important to prevent complications of
immobility. Nonpharmacologic methods in pain management may include physical,
cognitive-behavioral strategies, and lifestyle pain management. These methods are
used to provide comfort by altering psychological responses to pain.
41. Question
In planning pain reduction interventions, which pain theory provides information
most useful to nurses?

A. Specificity theory
B. Pattern theory
C. Gate-control theory
D. Central-control theory
Incorrect
Correct Answer: D. Central-control theory

No one theory explains all the factors underlying the pain experience, but the
central-control theory discusses brain opiates with analgesic properties and how
their release can be affected by actions initiated by the client and caregivers. In
central-control theory, the master control mechanism directs the muscle movement
based on linguistic goals. The gate-control, specificity, and patter theories do
not address pain control to the depth included in the central-control theory.

Option A: Specificity theory is one of the first modern theories for pain. It holds
that specific pain receptors transmit signals to a “pain center” in the brain that
produces the perception of pain. Von Frey (1895) argued that the body has a
separate sensory system for perceiving pain—just as it does for hearing and vision.
Option B: This theory ignored findings of specialized nerve endings and many of the
observations supporting the specificity and/or intensive theories of pain. The
theory stated that any somesthetic sensation occurred by a specific and particular
pattern of neural firing and that the spatial and temporal profile of firing of the
peripheral nerves encoded the stimulus type and intensity.
Option C: According to his theory, pain stimulation is carried by small, slow
fibers that enter the dorsal horn of the spinal cord; then other cells transmit the
impulses from the spinal cord up to the brain. These fibers are called T-cells. The
T-cells can be located in a specific area of the spinal cord, known as the
substantia gelatinosa. These fibers can have an impact on the smaller fibers that
carry the pain stimulation.
42. Question
Ryan underwent an open reduction and internal fixation of the left hip. One day
after the operation, the client is complaining of pain. Which data would cause the
nurse to refrain from administering the pain medication and to notify the health
care provider instead?

A. Left hip dressing dry and intact.


B. Blood pressure of 114/78 mm Hg; pulse rate of 82 beats per minute.
C. Left leg in functional anatomic position.
D. Left foot cold to touch; no palpable pedal pulse.
Incorrect
Correct Answer: D. Left foot cold to touch; no palpable pedal pulse.

A left foot cold to touch without palpable pedal pulse represents an abnormal
finding on neurovascular assessment of the left leg. The client is most likely
experiencing some complication from surgery, which requires immediate medical
intervention. The nurse should notify the health care provider of these findings.

Option A: A dry and intact hip dressing is a normal assessment of findings that do
not require medical intervention. A dressing is considered INTACT if portions of
the white dressing border have lifted from the skin as long as the clear viewing
window maintains full contact with the skin. The skin under the viewing window does
not appear visibly soiled with exudate or blood. The skin under the viewing window
does not appear dampened or moist with sweat, exudate, fluid, or blood.
Option B: A blood pressure of 114/78 mm Hg and pulse rate of 82 beats per minute
are normal assessment findings that do not require medical intervention. The normal
range used in an adult is between 60 to 100 beats /minute with rates above 100
beats/minute and rates and below 60 beats per minute, referred to as tachycardia
and bradycardia, respectively. The respiratory rate is the number of breaths per
minute. The normal breathing rate is about 12 to 20 beats per minute in an average
adult.
Option C: A left foot in functional anatomic position are all normal assessment
findings that do not require medical intervention. It functions as a rigid
structure for weight-bearing and it can also function as a flexible structure to
conform to uneven terrain.
43. Question
Which term would the nurse use to document pain at one site that is perceived in
another site?

A. Referred pain
B. Phantom pain
C. Intractable pain
D. Aftermath of pain
Incorrect
Correct Answer: A. Referred pain

Referred pain is pain occurring at one site that is perceived in another site.
Referred pain follows dermatome and nerve root patterns. Referred pain is pain
perceived at a location other than the site of the painful stimulus/ origin. It is
the result of a network of interconnecting sensory nerves that supply many
different tissues. When there is an injury at one site in the network it is
possible that when the signal is interpreted in the brain signals are experienced
in the surrounding nervous tissue.

Option B: Phantom pain refers to pain in a part of the body that is no longer
there, such as in amputation. Phantom pain is pain that feels like it’s coming from
a body part that’s no longer there. Doctors once believed this post-amputation
phenomenon was a psychological problem, but experts now recognize that these real
sensations originate in the spinal cord and brain.
Option C: Intractable pain refers to moderate to severe pain that cannot be
relieved by any known treatment. Intractable pain refers to a type of pain that
can’t be controlled with standard medical care. Intractable essentially means
difficult to treat or manage. This type of pain isn’t curable, so the focus of
treatment is to reduce the discomfort.
Option D: Aftermath of pain, a phase of the pain experience and the most neglected
phase address the client’s response to the pain experience. The complexity of pain
physiology makes some pains more difficult to manage than others. Acute
postoperative pain normally responds well to analgesia, but this should be
complemented by strategies such as comfortable positioning, distraction, TENS, and
reassurance.
44. Question
Chuck, who is in the hospital, complains of abdominal pain that ranks 9 on a scale
of 1 (no pain) to 10 (worst pain). Which interventions should the nurse implement?
Select all that apply.

A. Assessing the client's bowel sounds.


B. Taking the client's blood pressure and apical pulse.
C. Obtaining a pulse oximeter reading.
D. Notifying the health care provider.
E. Determining the last time the client received pain medication.
F. Encouraging the client to turn, cough, and deep breathe.
Incorrect
Correct Answers: A, B, & E
The nurse must rule out complications prior to administering pain medication, so
her interventions would include assessing to make sure the client has bowel sounds
and determining if the client is hemorrhaging by checking the client’s blood
pressure and pulse. The nurse must also make sure the pain medication is due
according to the health care provider’s orders. Obtaining a pulse oximeter reading
and turning, coughing, and deep breathing will not help the client’s pain.

Option A: Additionally, the nurse should ask the following questions during pain
assessment to determine its history: (1) effectiveness of previous pain treatment
or management; (2) what medications were taken and when; (3) other medications
being taken; (4) allergies or known side effects to medications.
Option B: Pain should be screened every time vital signs are evaluated. Many health
facilities set pain assessment as the “fifth vital sign” and should be added to
during routine vital signs assessment.
Option C: Investigate signs and symptoms related to pain. Bringing attention to
associated signs and symptoms may help the nurse in evaluating the pain. In some
instances, the existence of pain is disregarded by the patient.
Option D: There is no need to notify the health care provider in this situation.
Some patients may be satisfied when pain is no longer intense; others will demand
complete elimination of pain. This influences the perceptions of the effectiveness
of the treatment modality and their eagerness to engage in further treatments.
Option E: Some patients may be hesitant to try the effectiveness of
nonpharmacological methods and may be willing to try traditional pharmacological
methods (i.e., use of analgesics). A combination of both therapies may be more
effective and the nurse has the duty to inform the patient of the different methods
to manage pain.
Option F: Stress correlates to an increase in pain perception by increasing muscle
tension and activating the SNS. Eliciting a relaxation response decreases the
effects of stress on pain. Examples include directed meditation, music therapy,
deep breathing.
45. Question
Albert, who suffered severe burns 6 months ago, is expressing concern about the
possible loss of job-performance abilities and physical disfigurement. Which
intervention is the most appropriate for him?

A. Referring the client for counseling and occupational therapy.


B. Staying with the client as much as possible and building trust.
C. Providing cutaneous stimulation and pharmacologic therapy.
D. Providing distraction and guided imagery techniques.
Incorrect
Correct Answer: A. Referring the client for counseling and occupational therapy

Because it has been 6 months, the client needs professional help to get on with
life and handle the limitations imposed by the current problems. Staying with the
client, building trust, and providing methods of pain relief, such as cutaneous
stimulation, medications, distraction, and guided imagery interventions, would have
been more appropriate in earlier stages of postburn injury when physical pain was
most severe and fewer psychological factors needed to be addressed.

Option B: Explain to the patient what happened. Provide opportunities for questions
and give honest answers. Compassionate statements reflecting the reality of the
situation can help the patient and SO acknowledge that reality and begin to deal
with what has happened.
Option C: The burned patient may require around-the-clock medication and dose
titration. IV method is often used initially to maximize drug effect. Concerns of
patient addiction or doubts regarding the degree of pain experienced are not valid
during the emergent/acute phase of care, but narcotics should be decreased as soon
as feasible and alternative methods for pain relief initiated.
Option D: Encourage use of stress management techniques: progressive relaxation,
deep breathing, guided imagery, and visualization. Refocuses attention, promotes
relaxation, and enhances the sense of control, which may reduce pharmacological
dependency.
46. Question
Mrs. Bagapayo who had abdominal surgery 3 days earlier complains of sharp,
throbbing abdominal pain that ranks 8 on a scale of 1 (no pain) to 10 (worst pain).
Which intervention should the nurse implement first?

A. Assessing the client to rule out possible complications secondary to surgery.


B. Checking the client's chart to determine when pain medication was last
administered.
C. Explaining to the client that the pain should not be this severe 3 days
postoperatively.
D. Obtaining an order for a stronger pain medication because the client's pain has
increased.
Incorrect
Correct Answer: A. Assessing the client to rule out possible complications
secondary to surgery.

The nurse’s immediate action should be to assess the client in an attempt to


exclude possible complications that may be causing the client’s complaints. The
health care provider ordered the pain medication for routine postoperative pain
that is expected after abdominal surgery, not for such complications as hemorrhage,
infection, or dehiscence. The nurse should never administer pain medication without
assessing the client first.

Option B: Checking the client’s chart is appropriate after the nurse determines
that the client is not experiencing complications from surgery. It is essential to
assist patients to express as factually as possible (i.e., without the effect of
mood, emotion, or anxiety) the effect of pain relief measures. Inconsistencies
between behavior or appearance and what the patient says about pain relief (or lack
of it) may be more a reflection of other methods the patient is using to cope with
the pain rather than pain relief itself.
Option C: Pain is subjective, and each person has his own level of pain tolerance.
The nurse must always believe the client’s complaint of pain. Nurses have the duty
to ask their clients about their pain and believe their reports of pain.
Challenging or undermining their pain reports results in an unhealthy therapeutic
relationship that may hinder pain management and deteriorate rapport.
Option D: Obtaining an order for a strong medication may be appropriate after the
nurse assesses the client and checks the chart to see whether the current analgesic
is infective. The World Health Organization (WHO) in 1986 published guidelines in
the logical usage of analgesics to treat cancer using a three-step ladder approach
– also known as the analgesic ladder. The analgesic ladder focuses on aligning the
proper analgesics with the intensity of pain.
47. Question
Which term refers to the pain that has a slower onset, is diffuse, radiates, and is
marked by somatic pain from organs in any body activity?

A. Acute pain
B. Chronic pain
C. Superficial pain
D. Deep pain
Incorrect
Correct Answer: D. Deep pain

Deep pain has a slow onset, is diffuse, and radiates, and is marked by somatic pain
from organs in any body activity. Deep somatic pain originates from structures
deeper within the body, such as the joints, bones, tendons, and muscles. Like
visceral pain, deep somatic pain is usually dull and aching. Deep somatic pain can
either be experienced locally or more generally depending on the degree of trauma.

Option A: Acute pain is rapid in onset, usually temporary (less than 6 months), and
subsides spontaneously. Acute pain is a type of pain that typically lasts less than
3 to 6 months or pain that is directly related to soft tissue damage such as a
sprained ankle or a paper cut. Acute pain is of short duration but it gradually
resolves as the injured tissues heal.
Option B: Chronic pain is marked by gradual onset and lengthy duration (more than 6
months). Chronic pain is pain that is ongoing and usually lasts longer than six
months. This type of pain can continue even after the injury or illness that caused
it has healed or gone away. Pain signals remain active in the nervous system for
weeks, months, or years.
Option C: Superficial pain has an abrupt onset with sharp, stinging quality.
Superficial pain arises from nociceptive receptors in the skin and mucous
membranes. Superficial somatic pain is the type of pain that happens with common
everyday injuries and is characterized as pricking, sharp, burning, or throbbing
pain.
48. Question
A 50-year-old widower has arthritis and remains in bed too long because it hurts to
get started. Which intervention should the nurse plan?

A. Telling the client to strictly limit the amount of movement of his inflamed
joints.
B. Teaching the client's family how to transfer the client into a wheelchair.
C. Teaching the client the proper method for massaging inflamed, sore joints.
D. Encouraging gentle range-of-motion exercises after administering aspirin and
before rising.
Incorrect
Correct Answer: D. Encouraging gentle range-of-motion exercises after administering
aspirin and before rising.

Aspirin raises the pain threshold and, although range-of-motion exercises hurt,
mild exercise can relieve pain on rising. A tailored program that includes a
balance of three types of exercises — range-of-motion, strengthening, and endurance
— can relieve the symptoms of arthritis and protect joints from further damage.

Option A: Strict limitation of motion only increases the client’s pain. The
stiffness, pain, and swelling associated with arthritis can severely reduce the
range of motion of joints (the distance joints can move in certain directions).
Avoiding physical activity because of pain or discomfort also can lead to
significant muscle loss and excessive weight pain.
Option B: Having others transfer the client into a wheelchair does not increase his
feelings of dependency. Range-of-motion exercises (also called stretching or
flexibility exercises) help maintain normal joint function by increasing and
preserving joint mobility and flexibility.
Option C: Massage increases inflammation and should be avoided with this client.
During the course of a range-of-motion exercise program, the joints are stretched
progressively farther until the normal or near-normal range is achieved and
maintained.
49. Question
Which intervention should the nurse include as a nonpharmacologic pain-relief
intervention for chronic pain?

A. Referring the client for hypnosis.


B. Administering pain medication as prescribed.
C. Removing all glaring lights and excessive noise.
D. Using transcutaneous electric nerve stimulation.
Incorrect
Correct Answer: D. Using transcutaneous electric nerve stimulation.

Nonpharmacologic pain relief interventions include cutaneous stimulation, back


rubs, biofeedback, acupuncture, transcutaneous electric nerve stimulation, and
more. Transcutaneous electrical nerve stimulation (TENS) is a therapy that uses low
voltage electrical current to provide pain relief. A TENS unit consists of a
battery-powered device that delivers electrical impulses through electrodes placed
on the surface of your skin. The electrodes are placed at or near nerves where the
pain is located or at trigger points.

Option A: Hypnosis is considered an alternative therapy. Hypnosis is a set of


techniques designed to enhance concentration, minimize one’s usual distractions,
and heighten responsiveness to suggestions to alter one’s thoughts, feelings,
behavior, or physiological state.
Option B: Medications are pharmacologic measures. A wide range of drugs are used to
manage pain resulting from inflammation in response to tissue damage, chemical
agents/pathogens (nociceptive pain), or nerve damage (neuropathic pain).
Option C: Although removing glaring lights and excessive noise help to reduce or
remove noxious stimuli, it is not specific to pain relief. A noxious stimulus is
actually, or potentially, damaging to tissue and liable to cause pain, but does not
invariably do so. Some noxious stimuli, particularly in the viscera, do not cause
nociceptive responses.
50. Question
A 12-year-old student falls off the stairs, grabs his wrist, and cries, “Oh, my
wrist! Help! The pain is so sharp, I think I broke it.” Based on this data, the
pain the student is experiencing is caused by impulses traveling from receptors to
the spinal cord along which type of nerve fibers?

A. Type A-delta fibers


B. Autonomic nerve fibers
C. Type C fibers
D. Somatic efferent fibers
Incorrect
Correct Answer: A. Type A-delta fibers

Type A-delta fibers conduct impulses at a very rapid rate and are responsible for
transmitting acute sharp pain signals from the peripheral nerves to the spinal
cord. Only type A-delta fibers transmit sharp, piercing pain. They respond to
stimuli such as cold and pressure, and as nociceptors stimulation of them is
interpreted as fast/first pain information.

Option B: The autonomic system regulates involuntary vital functions and organ
control such as breathing. An autonomic nerve pathway involves two nerve cells. One
cell is located in the brainstem or spinal cord. It is connected by nerve fibers to
the other cell, which is located in a cluster of nerve cells (called an autonomic
ganglion). Nerve fibers from these ganglia connect with internal organs.
Option C: Type C fibers transmit sensory input at a much slower rate and produce a
slow, chronic type of pain. The C group fibers are unmyelinated and have a small
diameter and low conduction velocity, whereas Groups A and B are myelinated. Group
C fibers include postganglionic fibers in the autonomic nervous system (ANS), and
nerve fibers at the dorsal roots (IV fiber). These fibers carry sensory
information.
Option D: Somatic efferent fibers affect the voluntary movement of skeletal muscles
and joints. General somatic efferent fibers carry motor impulses to somatic
skeletal muscles. In the head, the tongue and extraocular muscles are of this type.
Cranial nerves III, IV, VI, and XII carry these fibers.
51. Question
Which nursing intervention takes the highest priority when caring for a newly
admitted client who’s receiving a blood transfusion?
A. Warming the blood prior to transfusion.
B. Informing the client that the transfusion usually takes 4 to 6 hours.
C. Documenting blood administration in the client chart.
D. Instructing the client to report any itching, chest pain, or dyspnea.
Incorrect
Correct Answer: D. Instructing the client to report any itching, headache, or
dyspnea.

This will help the nurse take immediate action in case a reaction happens during a
transfusion. There are multiple complications of blood transfusions, including
infections, hemolytic reactions, allergic reactions, transfusion-related lung
injury (TRALI), transfusion-associated circulatory overload, and electrolyte
imbalance.

Option A: There is no evidence that warming blood is beneficial to the patient when
transfusion is slow. At transfusion rates of greater than 100 mL/minute, cold blood
may be a contributing factor in cardiac arrest. However, keeping the patient warm
is probably more important than warming the blood.
Option B: Transfusion of a unit of blood should be completed within a maximum
period of four hours after removal from the blood fridge: discard the unit if this
period is exceeded. If blood has been out of the blood bank refrigerator for more
than 30 minutes and is not transfused, then the unit must be returned to the
laboratory, where it will be disposed of.
Option C: Documentation related to transfusion therapy should include verification
of the prescribed blood product and blood product compatibility; verification of
appropriate clinical indication for the transfusion; the date and time of
transfusion, type of blood product administered, in addition to the volume,
infusion rate, and time of initiation and completion of transfusion; any medication
administered, including premedication (if I.V. drugs are required during
transfusion, another I.V. site is required); the patient’s clinical status
throughout the transfusion therapy, including patient assessment data such as vital
signs and lung sounds; the patient’s response to therapy including any
complications or adverse reactions, treatment required, and response to that
treatment; and the amount of blood transfused and the return of the unused portion
to the blood bank.
52. Question
Nurse Paulo has received a blood unit from the blood bank and has rechecked the
blood bag properly with nurse Edward. Prior to the facilitation of the blood
transfusion, nurse Paulo priority checks which of the following?

A. Intake and output


B. NPO standing order
C. Vital signs
D. Skin turgor
Incorrect
Correct Answer: C. Vital signs

The nurse must assess the vital signs before and 15 minutes after the procedure so
that any changes during the transfusion may indicate a transfusion reaction is
happening. The nurse remains with the client, observing signs and symptoms and
monitoring vital signs as often as every 5 minutes.

Option A: Monitoring the intake and output during blood transfusion may be done,
but not as often as necessary. Monitoring of intake helps the caregiver to ensure
that the patient has a proper intake of fluid and other nutrients. Monitoring of
output helps determine whether there is an adequate output of urine as well as
normal defecation.
Option B: A patient on blood transfusion is not placed in an NPO standing order.
Current nil per os (NPO) standards promote pre-operative fasting as an approach to
reduce the volume and acidity of a patient’s stomach contents to reduce the risks
of regurgitation and subsequent pulmonary aspiration. Pre-anesthesia fasting
standards apply to any procedure where sedative medications reduce the protective
airway reflex that under normal conditions prevent aspiration.
Option D: Physical findings suggestive of volume depletion include dry mucous
membranes, decreased skin turgor, and low jugular venous distention. While the
incidence of hypovolemic shock from extracellular fluid loss is difficult to
quantify, it is known that hemorrhagic shock is most commonly due to trauma. In one
study, 62.2% of massive transfusions at a level 1 trauma center were due to
traumatic injury.
53. Question
A client is brought to the emergency department having experienced blood loss due
to a deep puncture wound. A 3 unit Fresh-frozen plasma (FFP) is ordered. The nurse
determines that the reason behind this order is to:

A. Provide clotting factors and volume expansion.


B. Increase hemoglobin, hematocrit, and neutrophil levels.
C. Treat platelet dysfunction.
D. Treat thrombocytopenia.
Incorrect
Correct Answer: A. Provide clotting factors and volume expansion.

Fresh-frozen plasma may be used to provide clotting factors or volume expansion. It


is rich in clotting factors and can be thawed quickly and transfused right away.
Fresh frozen plasma is the fluid portion of a unit of whole blood frozen in a
designated time frame, usually within 8 hours. FFP contains all coagulation factors
except platelets.

Option B: Increasing hemoglobin, hematocrit, and neutrophil levels is not an


indication for FFP. FFP corrects coagulopathy by replacing or supplying plasma
proteins in patients who are deficient in or have defective plasma proteins. A
standard dose of 10 to 20 mL/kg (4 to 6 units in adults) will raise factor levels
by approximately 20%.
Option C: FFP does not contain platelets. Other situations where the administration
of FFP cannot be recommended for or against based on systematic review include FFP
transfusion at a plasma-to-RBC ratio of 1:3 or more in trauma patients with massive
transfusion. Conditions that cause the deficiency of multiple coagulation factors
and may require the administration of FFP include liver disease and disseminated
intravascular coagulation.
Option D: Treating thrombocytopenia is incorrect since FFP does not contain any
platelet. FFP contains fibrinogen (400 to 900 mg/unit), albumin, protein C, protein
S, antithrombin, tissue factor pathway inhibitor. It is free of erythrocytes and
leukocytes. FFP provides some volume resuscitation, as each unit contains
approximately 250 ml.
54. Question
Nurse Amanda is caring for a client with severe blood loss who is prescribed
multiple transfusions of blood. Nurse Amanda obtains which most essential piece of
equipment to prevent the risk of cardiac dysrhythmias?

A. Cardiac monitor
B. Blood warmer
C. ECG machine
D. Infusion pump
Incorrect
Correct Answer: B. Blood warmer

Rapid transfusion of cool blood puts the client at risk for cardiac dysrhythmias.
Modern methods of very rapid transfusion in resuscitation would cause clinically
dangerous hypothermia if unmodified, ice-cold blood were to be so transfused. These
needs must be reconciled in the interest of adequate patient care–hence the need
for blood warming. Countercurrent in-line blood warmers and the method of rapid
warm saline admixture can both be used successfully for rapid, massive
transfusions.

Option A: Cardiac monitor is used to assess for any blood transfusion-related


complication, but they do not prevent the occurrence of cardiac dysrhythmia. During
the blood transfusion process, patients’ vital signs (heart rate, blood pressure,
temperature, and respiration rate) should be monitored throughout the procedure and
recorded. Follow the organization’s policy on how often the vital signs should be
measured.
Option C: ECG machine is used to assess for any blood transfusion-related
complication, but they do not prevent the occurrence of cardiac dysrhythmia. Many
severe reactions occur within the first 30 minutes of commencing a transfusion of a
blood component unit (SHOT 2008). Close observation during this period is
essential.
Option D: Infusion pump is not beneficial in this case since the infusion must be
given rapidly. SHOT 2008 recommends that patients be observed during the subsequent
24 hours because, on occasion, transfusion reactions can occur many hours after
transfusion is completed.
55. Question
A client is receiving a first-time blood transfusion of packed RBC. How long should
the nurse stay and monitor the client to ensure a transfusion reaction will not
happen?

A. 15 minutes
B. 30 minutes
C. 45 minutes
D. 60 minutes
Incorrect
Correct Answer: A. 15 minutes

Usually, a transfusion reaction occurs within 15 minutes of a transfusion. For each


unit of blood transfused, monitor the patient before starting the transfusion
(baseline observation; 15 minutes after starting the transfusion; at least every
hour during transfusion; and carry out a final set of observations 15 minutes after
each unit has been transfused.

Option B: Staying with the patient for 30 minutes might be too long. Acute
reactions may occur in 1% to 2% of transfused patients. Rapid recognition and
management of the reaction may save the patient’s life. Once immediate action has
been taken, careful and repeated clinical assessment is essential to identify and
treat the patient’s problems.
Option C: 45 minutes of staying and monitoring the patient for transfusion
reactions is too long. All suspected acute transfusion reactions should be reported
immediately to the blood transfusion center and to the doctor responsible for the
patient. With the exception of urticarial allergic reactions and febrile non-
hemolytic reactions, all are potentially fatal and require urgent treatment.
Option D: Most transfusion reactions occur during the first 15 minutes of
transfusion. 60 minutes is too long. However, transfusion-transmitted infections
are the serious delayed complications of transfusion. Since a delayed transfusion
reaction may occur days, weeks, or months after the transfusion, the association
with the transfusion may not be recognized.
56. Question
Nurse Rick is administering 2 unit-packed RBCs on a client with low hemoglobin. The
nurse will prepare which of the following in order to transfuse the blood?

A. Microfusion set
B. Polyvol Pro Burette Set
C. Photofusion set
D. Tubing with an in-line filter
Incorrect
Correct Answer: D. Tubing with an in-line filter

The in-line filter helps ensure that any particles larger than the size of the
filter are caught in the filter and are not infused with the client. Use a new,
sterile blood administration set containing an integral 170?200µ filter. Change the
set to at least 12?hourly during a blood transfusion. In a very warm climate,
change the set more frequently and usually after every four units of blood, if
given within a 12?hour period.

Option A: Microinfusion set is incorrect since the tubing that should be used is a
macro drip. Use a fresh blood administration set or special platelet transfusion
set, primed with saline. All blood components can be slowly infused through small?
bore cannulas or butterfly needles, e.g. 21 to 25 G. For rapid infusion, large?bore
cannulas, e.g. 14 G, are needed.
Option B: Polyvol Pro Burette set is used for administration of IV medication
infusion. The Polymed Polyvol Pro Latex Free Burette Set is a transparent, soft,
cylindrical, and calibrated measured volume chamber with a bold marking scale. It
has a micro dropper of 60 drops per ml. The burette set comes with a 15 ?m fluid
filter to reduce particulate matter.
Option C: Photofusion set is incorrect since blood does not need any protection
from light. It protects light-sensitive drugs from UV exposure & its harmful
effects. It has a 15-micron fluid filter in the drip chamber
57. Question
To verify the age of blood cells in blood, the nurse will check which of the
following?

A. Blood type
B. Blood group
C. Blood identification number
D. Blood expiration date
Incorrect
Correct Answer: D. Blood expiration date

The safe storage of blood usually takes 35 days. Examining the expiration date is
an important responsibility of a nurse prior to hanging the blood. Once issued by
the blood centre, the transfusion of whole blood, red cells, platelet concentrate
and thawed fresh frozen plasma should be commenced within 30 minutes of removal
from the optimal storage conditions.

Option A: A 5 mL blood sample should be collected into a dry test tube and then
correctly and clearly labeled with the patient’s details, and submitted to the
blood center for testing. The taking of a blood sample from the patient needs
supervision. If the patient is conscious at the time of taking the sample, ask
him/her to identify himself/herself by given name and all other appropriate
information.
Option B: When taking a blood sample for cross match, complete the whole procedure
before any other task is undertaken – it is important that there are no
interruptions during the process. When correctly performed, compatibility tests
will confirm ABO compatibility between component and recipient and will detect the
most clinically significant unexpected antibodies.
Option C: Always take a completed patient documentation label to the issue room of
the blood transfusion department when collecting the first unit of blood. Match the
details on the blood request form against the blood compatibility label (tag), the
bag unit number, and the patient documentation label. When receiving the unit of
blood in the clinical area, check that it is the right unit for the right patient.
58. Question
A client has an order to receive one unit of packed RBCs. The nurse makes sure
which of the following intravenous solutions to hang with the blood product at the
client’s bedside?

A. 0.9% sodium chloride.


B. 5% dextrose in 0.9% sodium chloride.
C. Balanced Multiple Maintenance Solution with 5% Dextrose.
D. 5% dextrose in 0.45% sodium chloride.
Incorrect
Correct Answer: A. 0.9% sodium chloride.

0.9% sodium chloride is a standard solution used to follow infusion of blood


products. Of the various intravenous solutions, only isotonic saline (0.9%) is
recommended for use with blood components. Other commonly used intravenous
solutions will cause varying degrees of difficulty when mixed with red cells.

Option B: 5% dextrose in water will hemolyze red cells. Intravenous solutions


containing calcium, such as Lactated Ringer’s solution, can cause clots to form in
the blood. Prior to blood transfusion, completely flush incompatible intravenous
solutions and drugs from the blood administration set with isotonic saline.
Option C: Normal saline is the only compatible solution to use with the blood or
blood component. Crystalloid solutions and medications may cause agglutination
and/or hemolysis of the blood or blood components.
Option D: IV solution containing dextrose in water will hemolyze red cells. Only
isotonic, calcium-free IV solutions should be added to, or come in contact with
blood products. Calcium may bind with the citrate anticoagulant and promote
clotting in the tubing. Excess glucose and/or dextrose causes hemolysis and
shortens red cell survival. Studies have shown other IV solutions to be compatible
with citrated blood components. However, these solutions should only be considered
compatible in situations where the use of 0.9% NaCl would lead to undesirable
metabolic abnormalities.
59. Question
Nurse Jay is caring for a client with an ongoing transfusion of packed RBCs when
suddenly the client is having difficulty breathing, skin is flushed, and having
chills. Which action should nurse Jay take first?

A. Administer oxygen.
B. Place the client on a droplight.
C. Check the client's temperature.
D. Stop the transfusion.
Incorrect
Correct Answer: D. Stop the transfusion.

The client in this situation is experiencing a transfusion reaction so the priority


action of the nurse is to first stop the transfusion. Disconnect the transfusion
set-but keep the IV line open with 0.9% saline to provide access for possible IV
drug infusion. Send the blood bag and tubing to the blood bank for repeat typing
and culture.

Option A: Place the client in Fowler’s position with shortness of breath and
administer O2 therapy. The nurse remains with the client, observing signs and
symptoms and monitoring vital signs as often as every 5 minutes. Obtain a urine
specimen and send it to the laboratory to determine presence of hemoglobin as a
result of RBC hemolysis.
Option B: Placing the client under a drop light would not manage his difficulty in
breathing. For circulatory overload, immediate treatment includes positioning the
patient upright with feet dependent; diuretics, oxygen, and aminophylline may be
prescribed. The nurse prepares to administer emergency drugs such as
antihistamines, vasopressor, fluids, and steroids as per the physician’s order or
protocol.
Option C: Febrile, nonhemolytic transfusion reactions are treated symptomatically
with antipyretics; leukocyte-poor blood products may be recommended for subsequent
transfusions. Blood container, tubing, attached label, and transfusion record are
saved and returned to the laboratory for analysis.
60. Question
After terminating the transfusion during a reaction, which action should the nurse
immediately be taken next?

A. Run a solution of 5% dextrose in water.


B. Run normal saline at a keep-vein-open rate.
C. Remove the IV line.
D. Fast drip 200ml normal saline.
Incorrect
Correct Answer: B. Run normal saline at a keep-vein-open rate.

The nurse will infuse normal saline at a KVO rate to keep the patency of the IV
line while waiting for further orders from the physician. A transfusion reaction
evaluation request form typically is used to document signs and symptoms of a
suspected reaction so that the blood bank can use this information, in conjunction
with laboratory testing, to arrive at a likely diagnosis. The blood bag, along with
the infusion set and anything else attached to the set, should be sent with the
transfusion reaction evaluation request.

Option A: IV solution containing dextrose will hemolyze the red cells. IV solution
containing dextrose in water will hemolyze red cells.Only isotonic, calcium-free IV
solutions should be added to, or come in contact with blood products. Calcium may
bind with the citrate anticoagulant and promote clotting in the tubing. Excess
glucose and/or dextrose causes hemolysis and shortens red cell survival.
Option C: The nurse will not remove the IV line because then there would be no IV
access route. Transfusion reaction treatment varies with the reaction.
Diphenhydramine and acetaminophen are some of the most commonly used drugs for
treating mild allergic and febrile nonhemolytic reactions. For other reactions,
expert consultation should be considered. In cases of acute hemolytic reaction,
baseline laboratory tests should be performed and urine should be kept flowing,
possibly with alkalinization.
Option D: Doing a fast drip will potentially lead to congestion and is not done
without the physician’s order. Volume overload may require diuretics. TRALI is
treated with oxygen and supportive care, which may involve intubation. Bacterial
contamination may involve blood pressure support and antibiotics. Because
anaphylaxis is treated emergently according to hospital protocol, usually with
epinephrine and diphenhydramine, there may not be time for consultation until after
the patient is stabilized.
61. Question
A client is receiving a platelet transfusion. The nurse determines that the client
is gaining from this therapy if the client exhibits which of the following?

A. Less frequent febrile episodes.


B. Increased level of hematocrit.
C. Less episodes of bleeding.
D. Increased level of hemoglobin.
Incorrect
Correct Answer: C. Less episodes of bleeding.

Platelet transfusions may be given to prevent bleeding when the platelet count
falls down. In a study of bleeding risks in thrombocytopenic patients, Webert et
al. noted that the majority of severe bleeds were preceded by bleeds of lesser
severity. Even patients with petechiae (WHO grade 1 bleeding) were 2.5 times more
likely to experience clinically significant bleeding on the next day; patients
experiencing WHO grade 1 or 2 bleeding were three times more likely to have a
severe bleed the next day.

Option A: A decline in the febrile episode will happen after the transfusion of
agranulocytes. Transfusions of granulocytes have a long history of usage in
clinical practice to support and treat severe infection in high?risk groups of
patients with neutropenia or neutrophil dysfunction.
Option B: An increased level of hematocrit occurs after infusion of red blood
cells. Blood transfusion is used to treat acute anemia with the goal of increasing
blood oxygen-carrying capacity as determined by hematocrit (Hct), and oxygen
delivery (DO2).
Option D: An increased level of hemoglobin will happen after the transfusion of red
blood cells. The transfusion of red cell concentrates (RCC) is indicated in order
to achieve a fast increase in the supply of oxygen to the tissues, when the
concentration of hemoglobin (Hb) is low and/or the oxygen-carrying capacity is
reduced, in the presence of inadequate physiological mechanisms of compensation.
62. Question
Nurse Daniel is caring for a client receiving a transfusion of packed red blood
cells (PRBCs). The client started to vomit and to be nauseous. Client’s blood
pressure is 95/40 mm Hg from a baseline of 110/70 mm Hg. The client’s temperature
is 100.5°F orally from a baseline of 99.5°F orally. The nurse understands that the
client may be experiencing which of the following?

A. Circulatory overload
B. Delayed transfusion reaction
C. Hypocalcemia
D. Septicemia
Incorrect
Correct Answer: D. Septicemia

Septicemia happens with the transfusion of blood that is contaminated with


microorganisms. Assessment includes the rapid onset of high fever and chills,
hypotension, nausea, diarrhea, vomiting, and shock. Fever and/or chills are most
commonly associated with a febrile, non-hemolytic reaction, however; they can also
be the first sign of a more serious acute hemolytic reaction, TRALI, or septic
transfusion reaction. If the temperature rises 1 C or higher from the temperature
at the start of the transfusion, the transfusion should be stopped.

Option A: Circulatory overload causes hypertension, cough, dyspnea, chest pain,


tachycardia, and wheezing upon auscultation. Dyspnea, or shortness of breath, is a
concerning sign that can often be seen with more severe reactions including
anaphylaxis, TRALI, and TACO. It can also be seen by itself without accompanying
symptoms.
Option B: Delayed reaction can occur days to years after a transfusion. It causes
fever, rashes, mild jaundice, and oliguria or anuria. Typically caused by an
anamnestic response to a foreign antigen that the patient was previously exposed to
(generally by prior transfusion or pregnancy).
Option C: Hypocalcemia causes paresthesias, tetany, muscle cramps, hyperactive
reflexes, positive Trousseau’s sign, and positive Chovstek’s sign. Hypocalcemia is
said to be present when the total serum calcium concentration is less than 8.8
mg/dl. The disorder may be acquired or inherited but its presentation can vary-
from asymptomatic to life-threatening. Hypocalcemia is commonly seen in
hospitalized patients and for the most part, is mild in nature and only requires
supportive treatment.
63. Question
Packed red blood cells have been prescribed for a client with low hemoglobin and
hematocrit levels. The nurse takes the client’s temperature before hanging the
blood transfusion and records 100.8 °F. Which action should the nurse take?
A. Give an antipyretic and begin the transfusion.
B. Proceed with the transfusion.
C. Administer an antihistamine and begin the transfusion.
D. Delay hanging the blood and inform the physician.
Incorrect
Correct Answer: D. Delay hanging the blood and inform the physician.

If the patient has a temperature higher than 100 ° F, the unit of blood should be
hung and delayed until the physician is notified and has the opportunity to give
further orders. Fever and/or chills are most commonly associated with a febrile,
non-hemolytic reaction, however; they can also be the first sign of a more serious
acute hemolytic reaction, TRALI, or septic transfusion reaction.

Option A: Giving an antipyretic would require a physician’s order. All cases of


suspected reactions should prompt immediate discontinuation of the transfusion and
notification of the blood bank and treating clinician. A clerical check should be
performed by examining the product bag and confirming the patient’s identification.
The patient’s vital signs should be monitored and recorded at 15-minute intervals.
Option C: Administering an antihistamine is incorrect since the administration of
the medicine will need the physician’s prescription. Treatment of specific
transfusion reactions is most often supportive. For example, antihistamines (such
as diphenhydramine) can be given for a mild allergic reaction, or an antipyretic
can be given for a nonhemolytic febrile transfusion reaction.
Option B: The decision to administer the blood is not within the scope of nurse
practice. Currently, guidelines for transfusion of red blood cells (RBC), generally
follow a restrictive threshold. While there is some variation in the number for the
threshold, 7 g/dL is an agreed-upon value for asymptomatic healthy patients.
64. Question
A nurse is caring for a client requiring surgery and is ordered to have a standby
blood secured if in case a blood transfusion is needed during or after the
procedure. The nurse suggests to the client to do which of the following to lessen
the risk of possible transfusion reaction?

A. Request that any donated blood be screened twice by the blood bank.
B. Take iron supplements prior to the surgery and eat green leafy vegetables.
C. Do an autologous blood donation.
D. Have a family member donate their own blood.
Incorrect
Correct Answer: C. Do an autologous blood donation.

A donation of your own blood is autologous. Doing this will prevent the risk of
transfusion reaction. Autologous blood transfusion is the collection of blood from
a single patient and retransfusion back to the same patient when required. This is
in contrast to allogeneic blood transfusion where blood from unrelated/anonymous
donors is transfused to the recipient. The primary driving forces for the use of
autologous blood transfusion are to reduce the risk of transmission of infection
and to protect an increasingly scarce resource.

Option A: More recently, concerns have focussed on the blood-borne transmission of


variant Creutzfeldt–Jakob disease (vCJD). In 2004, case reports emerged of presumed
transmission of vCJD via allogeneic blood transfusion. Unlike hepatitis and HIV,
there is no effective screening test and the disease has a variable and often
prolonged asymptomatic incubation period.
Option B: As oral iron supplementation requires a significant amount of time, when
the interval before surgery is sufficient (at least 6–8 weeks) and no
contraindications are present, supplementation with oral iron and nutritional
advice may be appropriate for mild-to-moderate IDA and/or nonanemic ID or
insufficient iron stores.
Option D: Allogeneic donor blood is becoming an increasingly costly and scarce
resource. As demand for blood is outstripping donation, there is a real social and
economic pressure to increase the proportion of blood transfused by autologous
transfusion.
65. Question
A client is receiving a transfusion of one unit of cryoprecipitate. The nurse will
review which of the following laboratory studies to assess the effectiveness of the
therapy?

A. Serum electrolytes
B. White blood cell count
C. Coagulation studies
D. Hematocrit count
Incorrect
Correct Answer: C. Coagulation studies

The evaluation of the effective response of a cryoprecipitate transfusion is


assessed by monitoring coagulation studies and fibrinogen levels. Cryoprecipitate
Antihemophilic Factor, also called cryo, is a portion of plasma, the liquid part of
the blood. Cryo is rich in clotting factors, which are proteins that can reduce
blood loss by helping to slow or stop bleeding.

Option A: Crystalloids are the fluids of choice for most minor procedures. They are
sterile aqueous solutions that may contain glucose, various electrolytes, organic
salts, and nonionic compounds. Some examples of these solutes are sodium chloride,
potassium chloride, sodium bicarbonate, calcium carbonate, sodium acetate, sodium
lactate, and sodium gluconate.
Option B: White blood cells are transfused to treat life-threatening infections in
people who have a greatly reduced number of white blood cells or whose white blood
cells are functioning abnormally. The use of white blood cell transfusions is rare
because improved antibiotics and the use of cytokine growth factors that stimulate
people to produce more of their own white blood cells have greatly reduced the need
for such transfusions.
Option D: The average increase in hematocrit per liter of packed red blood cells
transfused was 6.4% +/- 4.1%. If 1 “unit” of packed red blood cells is
approximately 300 mL, this becomes a change of hematocrit of 1.9% +/- 1.2% per
“unit” of blood. The accepted correlation of about 1 “unit” of blood loss per 3%
change in hematocrit would be valid for a 500-cc unit, but a typical unit of packed
red blood cells is typically 300 cc.
66. Question
A nurse is teaching a client with pancreatitis about following a low-fat diet. The
nurse develops a list of high-fat foods to avoid and includes which food on the
item list?

A. Chocolate milk
B. Broccoli
C. Apple
D. Salmon
Incorrect
Correct Answer: A. Chocolate milk

Chocolate milk is a high-fat food. The pancreas helps with fat digestion, so foods
with more fat make the pancreas work harder. Registered dietitian Deborah Gerszberg
recommends that people with chronic pancreatitis limit their intake of refined
carbohydrates, such as white bread and high sugar foods. Refined carbohydrates can
lead to the pancreas releasing large amounts of insulin. Foods that are high in
sugar can also raise triglycerides.

Option B: Vegetables are low in fat because they do not come from animal sources.
Vegetables, beans, lentils, and whole grains are beneficial because of their fiber
content. Eating more fiber can lower the chances of having gallstones or elevated
levels of fats in the blood called triglycerides. Both of those conditions are
common causes of acute pancreatitis.
Option C: Fruits are low in fat because they do not come from animal sources.
Fruits are recommended for people with pancreatitis because they tend to be
naturally low in fat, which eases the amount of work the pancreas needs to do to
aid digestion.
Option D: Salmon is naturally lower in fat. Many types of fish, such as salmon,
lake trout, tuna, and herring, provide healthy omega-3 fat. But avoid fish canned
in oil, such as sardines in olive oil. Bake, broil, or grill meats, poultry, or
fish instead of frying them in butter or fat.
67. Question
The nurse is giving dietary instructions to a client who is on a vegan diet. The
nurse provides dietary teaching focus on foods high in which vitamin that may be
lacking in a vegan diet?

A. Vitamin A
B. Vitamin D
C. Vitamin E
D. Vitamin C
Incorrect
Correct Answer: B. Vitamin D

Deficiencies in vegetarian diets include vitamin B12 which is found in animal


products and vitamin D (if limited exposure to sunlight). Vegans and other
vegetarians who limit their intake of animal products may be at greater risk of
vitamin D deficiency than nonvegetarians because foods providing the highest amount
of vitamin D per gram naturally are all from animal sources, and fortification with
vitamin D currently occurs in few foods.

Option A: Plant sources contain vitamin A in the form of carotenoids which have to
be converted during digestion into retinol before the body can use it. Carotenoids
are the pigments that give plants their green color and some fruits and vegetables
their red or orange color.
Option C: The best way to get the daily requirement of vitamin E is by eating food
sources. Vitamin E is found in vegetable oils, nuts, seeds, green leafy vegetables,
and fortified breakfast cereals. It is an antioxidant. This means it protects body
tissue from damage caused by substances called free radicals. Free radicals can
harm cells, tissues, and organs. They are believed to play a role in certain
conditions related to aging.
Option D: Vitamin C can be found in fruits and vegetables, which are eaten by a
vegetarian. Humans are unable to synthesize vitamin C, so it is strictly obtained
through the dietary intake of fruits and vegetables. Citrus fruits, berries,
tomatoes, potatoes, and green leafy vegetables are excellent sources of vitamin C.
68. Question
A nurse is caring for a client with Wernicke-Korsakoff syndrome. The physician asks
the nurse to teach the client to consume thiamine-rich food. The nurse instructs
the client to increase the intake of which food items?

A. Chicken
B. Milk
C. Beef
D. Broccoli
Incorrect
Correct Answer: C. Beef

Food sources of thiamin include beef, liver, nuts, oats, oranges, pork, eggs,
seeds, legumes, peas, and yeast. In meat, the liver has the highest amount of
thiamine. Whereas three ounces of beefsteak gives 7% of the daily value of
thiamine, one serving of beef liver will give about 10%. One serving of cooked
salmon gives 18% of the daily value of thiamine.

Option A: Poultry contains niacin. Chicken meat, particularly chicken breast, is an


excellent source of protein as well as niacin. A three-ounce serving of skinless
breast meat provides 10.3 mg. Niacin is an essential nutrient that we mainly need
to get from foods. The body may also convert some tryptophan, one of the body’s
amino acids, into a nutrient.
Option B: Milk contains vitamins A, D, and B2. Milk contains the fat-soluble
vitamins A, D, E, and K. The content level of fat-soluble vitamins in dairy
products depends on the fat content of the product. Milk contains the water-soluble
vitamins thiamin (vitamin B1), riboflavin (vitamin B2), niacin (vitamin B3),
pantothenic acid (vitamin B5), vitamin B6 (pyridoxine), vitamin B12 (cobalamin),
vitamin C, and folate. Milk is a good source of thiamin, riboflavin, and vitamin
B12.
Option D: Broccoli contains folic acid, vitamins C, E, and K. Broccoli is a good
source of fiber and protein and contains iron, potassium, calcium, selenium, and
magnesium as well as the vitamins A, C, E, K, and a good array of B vitamins
including folic acid.
69. Question
A client who is recovering from surgery has been ordered a change from a clear
liquid diet to a full liquid diet. The nurse would offer which full liquid item to
the client?

A. Popsicle
B. Carbonated beverages
C. Gelatin
D. Custard
Incorrect
Correct Answer: D. Custard

Full liquid food items include items such as plain ice cream, sherbet, breakfast
drinks, milk, pudding, and custard, soups that are strained, refined cooked
cereals, and strained vegetable juices. A full liquid diet is made up only of
fluids and foods that are normally liquid and foods that turn to liquid when they
are at room temperature, like ice cream.

Option A: A clear liquid diet is a specific dietary plan that only includes liquids
that are fully transparent at room temperature. Some items that may be allowed
include water, ice, fruit juices without pulp, sports drinks, carbonated drinks,
gelatin, tea, coffee, clear broths, and clear ice pops.
Option B: Carbonated beverages are part of a clear liquid diet. Items can have
color as long as they are transparent. Items such as milk and orange juice are not
considered clear liquids because they are not fully transparent and may take more
effort for the digestive system to break down, whereas grape juice is allowed (it
is pigmented, but fully transparent).
Option C: Gelatin is a clear liquid diet. The clear liquid diet assists in
maintaining hydration, provides electrolytes and calories, and offers some level of
satiety when a full diet is not appropriate, but may struggle to provide adequate
caloric needs if employed for more than five days.
70. Question
A postoperative client has been placed on a clear liquid diet. The nurse provides
the client with which items are allowed to be consumed on this diet?

A. Vegetable juices
B. Custard
C. Sherbet
D. Bouillon
Incorrect
Correct Answer: D. Bouillon

A clear liquid diet consists of foods that are relatively transparent to light and
liquid at room and body temperature. Foods allowed on the clear liquid diet
(bouillon, popsicles, plain gelatin, ice chips, sweetened tea or coffee (no
creamer), carbonated beverages, and water). The clear liquid diet assists in
maintaining hydration, it provides electrolytes and calories, and offers some level
of satiety when a full diet is not appropriate, but may struggle to provide
adequate caloric needs if employed for more than five days

Option A: Vegetable juices are part of a full liquid diet. A patient prescribed a
full liquid diet follows a specific diet type requiring all liquids and semi-
liquids but no forms of solid intake. Unlike a clear liquid diet, which includes
only liquids and semi-liquids that are non-opaque, a full liquid diet is more
inclusive, as it allows all types of liquids.
Option B: Custard is a full liquid diet. Patients not ready for a regular diet due
to elective or emergent procedures or who experience irregularity in
gastrointestinal function, dysphagia, a transition from prolonged fasted periods,
etc., are typically placed on a restrictive diet.
Option C: Sherbet is a full liquid diet. Dietary restrictions can be as restrictive
as no food or liquids allowed by mouth, which may increase in a stepwise fashion
until reaching regular nutrition. One step in that progression is a full liquid
diet.
71. Question
The nurse is teaching a client who has iron-deficiency anemia about foods she
should include in her diet. The nurse determines that the client understands the
dietary instructions if she selects which of the following from her menu?

A. Nuts and fish.


B. Oranges and dark green leafy vegetables.
C. Butter and margarine.
D. Sugar and candy.
Incorrect
Correct Answer: B. Oranges and dark green leafy vegetables.

Dark green leafy vegetables are rich in iron while oranges are a good source of
vitamin C, which enhances iron absorption. A diet plan for anemia needs to include
a healthful balance of iron-rich foods, such as leafy vegetables, lean meat, nuts
and seeds, and fortified breakfast cereals. It is also crucial to include foods
that can improve the body’s absorption of iron and avoid foods that may interfere
with this process.

Option A: Phytates also termed phytic acid is present in legumes, whole grains,
nuts, and brown rice. The phytic acid binds with the iron present in the digestive
tract and inhibits its absorption. Hence, anemic patients must avoid foods
containing phytates.
Option C: The mineral hinders iron absorption and therefore consuming calcium-
containing food products in combination with other iron-rich foods can affect how
much iron is being absorbed by the body. Dairy foods like milk, yoghurt, and cheese
should be avoided for this reason. Therefore, it is advisable to take calcium-
containing foods at different time slots.
Option D: The effect of the iron-chelating sugars, fructose, glucose, and
galactose, on iron absorption in rats has been examined. Fructose has an effect on
increasing iron absorption. Glucose and galactose have no effect on iron
absorption. These findings suggest that the metabolism of fructose is responsible
for changing iron absorption in the rat since it is metabolized during its
absorption, while glucose and galactose are not.
72. Question
The nurse instructs a client with renal failure who is receiving hemodialysis about
dietary modifications. The nurse determines that the client understands these
dietary modifications if the client selects which items from the dietary menu?

A. Mushroom and blueberry.


B. Beans and bananas.
C. Fish and tomato juice.
D. Potato and spinach.
Incorrect
Correct Answer: A. Mushroom and blueberry.

A renal diet is one that is low in sodium, phosphorus, potassium and protein. A
renal diet also emphasizes the importance of consuming high-quality protein and
usually limiting fluids. Some patients may also need to limit potassium and
calcium. Every person’s body is different, and therefore, it is crucial that each
patient works with a renal dietitian to come up with a diet that is tailored to the
patient’s needs.

Option B: Bananas are rich in potassium. The kidneys help to keep the right amount
of potassium in the body and they expel excess amounts into the urine. When the
kidneys fail, they can no longer remove excess potassium, so potassium levels build
up in the body.
Option C: Tomato juice is high in sodium. Too much sodium can be harmful for people
with kidney disease because their kidneys cannot adequately eliminate excess sodium
and fluid from the body. Processed foods often contain higher levels of sodium due
to added salt.
Option D: Potatoes and spinach are high in potassium. Potassium plays a role in
keeping the heartbeat regular and the muscles working correctly. Potassium is also
necessary for maintaining fluid and electrolyte balance in the bloodstream. When
the kidneys fail, they can no longer remove excess potassium, so potassium levels
build up in the body.
73. Question
A client with heart failure has been told to maintain a low sodium diet. A nurse
who is teaching this client about foods that are allowed includes which food item
in a list provided to the client?

A. Pretzels
B. Whole wheat bread
C. Tomato juice canned
D. Dried apricot
Incorrect
Correct Answer: D. Dried apricot

Foods that are lower in sodium include fruits and vegetables like dried apricot.
Dried apricots are sodium-free. Dried apricots, as part of a low sodium diet, may
reduce the risk of high blood pressure. Apricots contain numerous antioxidants,
most notably flavonoids. They help protect the body from oxidative stress, which is
linked to many chronic diseases.

Option A: These classic snacks are high in sodium — almost 20 percent of the
recommended daily intake is in one serving of pretzels. Too much sodium leads to
increased water retention, which can lead to bloating and puffiness, and too much
sodium over time can lead to heart disease.
Option B: Sodium is finding its way into a lot of whole wheat bread brands in
amounts that average 240 to 400 mg per slice. If your serving usually contains two
slices, the sodium can add up quickly.
Option C: Many tomato juice products contain added salt — which bumps up the sodium
content. For example, a 1.4-cup (340-ml) serving of Campbell’s 100% tomato juice
contains 980 mg of sodium — which is 43% of the DV. Research shows that diets high
in sodium may contribute to high blood pressure.
74. Question
The nurse is instructing a client with hyperkalemia on the importance of choosing
foods low in potassium. The nurse should teach the client to limit which of the
following foods?

A. Grapes
B. Carrot
C. Green beans
D. Lettuce
Incorrect
Correct Answer: B. Carrot

Carrots have 320 mg of potassium per 100 mg serving; green beans give 209 mg of
potassium, 194 mg for lettuce, and 191 mg for grapes all in 100 mg serving. Other
foods that are low in potassium include applesauce, blueberries, pineapple, and
cabbage. To minimize potassium buildup, a person with chronic kidney disease should
stick to a low-potassium diet of between 1,500 and 2,000 milligrams (mg) per day.
Limiting phosphorus, sodium, and fluids may also be important for people with
kidney dysfunction.

Option A: Grapes are also rich in potassium, but not as much as in carrots. They’re
also a good source of vitamin C, an essential nutrient and powerful antioxidant
necessary for connective tissue health. Grapes are high in a number of powerful
antioxidant compounds. In fact, over 1,600 beneficial plant compounds have been
identified in this fruit.
Option C: Half a cup of freshly cooked green beans has only 90 milligrams of
potassium and 18 milligrams of phosphorus, making them a great vegetable choice for
the kidney diet.
Option D: Lettuce is a popular vegetable and is usually eaten raw in salads.
Because CKD patients with hyperkalemia need to limit potassium intake from meals,
they are not able to eat large quantities of raw vegetables such as lettuce.
75. Question
A client is recovering from debridement of the right leg. A nurse encourages the
client to eat which food item that is naturally high in vitamin C to promote wound
healing?

A. Milk
B. Chicken
C. Banana
D. Strawberries
Incorrect
Correct Answer: D. Strawberries

Citrus fruits and juices are especially high in vitamin C. Strawberries are an
excellent source of vitamin C and manganese and also contain decent amounts of
folate (vitamin B9) and potassium. Strawberries are very rich in antioxidants and
plant compounds, which may have benefits for heart health and blood sugar control

Option A: Dairy products such as milk are high in vitamin B. Milk and other dairy
products pack about a third of the daily riboflavin requirement in just 1 cup (240
ml). Milk is also a good source of well-absorbed B12. Like other animal products,
milk also is a good source of B12, supplying 18% of the RDI per 1-cup (240-ml)
serving.
Option B: Meats such as chicken are high in vitamin B. Chicken and turkey,
especially the white meat portions, are high in B3 and B6. Poultry also supplies
smaller amounts of riboflavin, pantothenic acid, and cobalamin. Most of the
nutrients are in the meat, not the skin.
Option C: Bananas are rich in potassium. Bananas are rich in the mineral potassium.
Potassium helps maintain fluid levels in the body and regulates the movement of
nutrients and waste products in and out of cells. One medium-sized banana contains
422 milligrams (mg) of potassium.

You might also like